Buy BOOKS at Discounted Price

Arithmetic Progression

Class 10th Mathematics RS Aggarwal Solution
Exercise 11a
  1. 9, 15, 21, 27, ... Show that each of the progressions given below is an AP.…
  2. 11, 6, 1, - 4, ... Show that each of the progressions given below is an AP.…
  3. -1 , -5/6 , -2/3 , -1/2 , l Show that each of the progressions given below is…
  4. √2, √8, √18, √32,… Show that each of the progressions given below is an AP.…
  5. √20, √45, √80, √125,… Show that each of the progressions given below is an AP.…
  6. the 20th term of the AP 9, 13, 17, 21, ... . Find:
  7. the 35th term of the AP 20, 17, 14, 11, . Find:
  8. the 18th term of the AP √2, √18, √50, √98…. Find:
  9. the 9th term of the AP 3/4 , 5/4 , 7/4 , 9/4 l Find:
  10. the 15th term of the AP - 40, - 15, 10, 35,… Find:
  11. Find the 37th term of the AP 6 , 7 3/4 , 9 1/2 , 11 1/4 , l
  12. Find the 25th term of the AP 5 , 4 1/2 , 4 , 3 1/2 , 3 l
  13. 5, 11, 17, 23, ... Find the nth term of each of the following AP’s:…
  14. 16, 9, 2, - 5, ... Find the nth term of each of the following AP’s:…
  15. If the nth term of a progression is (4n - 10) show that it is an AP. Find its…
  16. How many terms are there in the AP 6, 10, 14, 18, ... , 174?
  17. How many terms are there in the AP 41, 38, 35, ..., 8?
  18. How many terms are there in the AP 18, 15 1/2 , 13, ..., - 47?
  19. Which term of the AP 3, 8, 13, 18, ... is 88?
  20. Which term of the AP 72, 68, 64, 60, ... is 0?
  21. Which term of the AP 5/6 , 1 , 1 1/3 , 1 1/3 , l is 3?
  22. Which term of the AP 21, 18, 15, ... is - 81?
  23. Which term of the AP 3, 8, 13, 18, ... will be 55 more than its 20th term?…
  24. Which term of the AP 5, 15, 25, ... will be 130 more than its 31st term?…
  25. If the 10th term of an AP is 52 and 17th term is 20 more than its 13th term,…
  26. Find the middle term of the AP 6, 13, 20, ..., 216.
  27. Find the middle term of the AP 10, 7, 4, ..., (-62).
  28. Find the sum of two middle most terms of the AP - 4/3 ,-1 , -2/3 , l 4 1/3…
  29. Find the 8th term from the end of the AP 7, 10, 13, ..., 184.
  30. Find the 6th term from the end of the AP 17, 14, 11, ..., (-40).
  31. Is 184 a term of the AP 3, 7, 11, 15, ...?
  32. Is - 150 a term of the AP 11, 8, 5, 2, ...?
  33. Which term of the AP 121, 117, 113, ... is its first negative term?…
  34. Which term of the AP 20 , 19 1/4 , 18 1/2 , 17 3/4 l is its first negative…
  35. The 7th term of an AP is - 4 and its 13th term is - 16. Find the AP.…
  36. The 4th term of an AP is zero. Prove that its 25th term is triple its 11th…
  37. The 8th term of an AP is zero. Prove that its 38th term is triple its 18th…
  38. The 4th term of an AP is 11. The sum of the 5th and 7th terms of this AP is…
  39. The 9th term of an AP is - 32 and the sum of its 11th and 13th terms is - 94.…
  40. Determine the nth term of the AP whose 7th term is - 1 and 16th term is 17.…
  41. If 4 times the 4th term of an AP is equal to 18 times its 18th term then find…
  42. If 10 times the 10th term of an AP is equal to 15 times the 15th term, show…
  43. Find the common difference of an AP whose first term is 5 and the sum of its…
  44. The sum of the 2nd and the 7th term of an AP is 30. If its 15th term is 1 less…
  45. For what value of n, the nth terms of the arithmetic progressions 63, 65, 67,…
  46. The 17th term of AP is 5 more than twice its 8th term. If the 11th term of the…
  47. The 24th term of an AP is twice its 10th term. Show that its 72nd term is 4…
  48. The 19th term of an AP is equal to 3 times its 6th term. If its 9th term is…
  49. If the pth term of an AP is q and its qth term is p then show that its (p +…
  50. The first and last terms of an AP are a and 1 respectively. Show that the sum…
  51. How many two - digit numbers are divisible by 6?
  52. How many two - digit numbers are divisible by 3?
  53. How many three - digit numbers are divisible by 9?
  54. How many numbers are there between 101 and 999, which are divisible by both 2…
  55. In a flower bed, there are 43 rose plants in the first row, 41 in the second,…
  56. A sum of Rs. 2800 is to be used to award four prizes. If each prize after the…
Exercise 11b
  1. Determine k so that (3k - 2), (4k - 6) and (k + 2) are three consecutive terms…
  2. Find the value of x for which the numbers (5x + 2), (4x - 1) and (x + 2) are in…
  3. If (3y - 1), (3y + 5) and (5y + 1) are three consecutive terms of an AP then…
  4. Find the value of x for which (x + 2), 2x, (2x + 3) are three consecutive terms…
  5. Show that (a - b)^2 , (a^2 + b^2) and (a + b)^2 are in AP.
  6. Find three numbers in AP whose sum is 15 and product is 80.
  7. The sum of three numbers in AP is 3 and their product is - 35. Find the…
  8. Divide 24 in three parts such that they are in AP and their product is 440.…
  9. The sum of three consecutive terms of an AP is 21 and the sum of the squares of…
  10. The angles of a quadrilateral are in AP whose common difference is 10°. Find…
  11. Find four numbers in AP whose sum is 28 and the sum of whose squares is 216.…
  12. Divide 32 into four parts which are the four terms of an AP such that the…
  13. The sum of first three terms of an AP is 48. If the product of first and…
Exercise 11c
  1. The first three terms of an AP are respectively (3y - 1), (3y + 5) and (5y +…
  2. If k, (2k - 1) and (2k + 1) are the three successive terms of an AP, find the…
  3. If 18, a, (b - 3) are in AP, then find the value of (2a - b).
  4. If the numbers a, 9, b, 25 form an AP, find a and b.
  5. If the numbers (2n - 1), (3n + 2) and (6n - 1) are in AP, find the value of n…
  6. How many three - digit natural numbers are divisible by 7?
  7. How many three - digit natural numbers are divisible by 9?
  8. If the sum of first m terms of an AP is (2m^2 + 3m) then what is its second…
  9. What is the sum of first n terms of the AP a, 3a, 5a, ....
  10. What is the 5th term from the end of the AP 2, 7, 12, ..., 47?
  11. If an denotes the nth term of the AP 2, 7, 12, 17, ..., find the value of (a30…
  12. The nth term of an AP is (3n + 5). Find its common difference.
  13. The nth term of an AP is (7 - 4n). Find its common difference.
  14. Write the next term of the AP
  15. Write the next term of the AP root 2 , root 8 , root 18 , l
  16. Which term of the AP 21, 18, 15, ... is zero?
  17. Find the sum of first n natural numbers.
  18. Find the sum of first n even natural numbers.
  19. The first term of an AP is p and its common difference is q. Find its 10th…
  20. If 4/5, a, 2 are in AP, find the value of a.
  21. If (2p + 1), 13, (5p - 3) are in AP, find the value of p.
  22. If (2p - 1), 7, 3p are in AP, find the value of p.
  23. If the sum of first p terms of an AP is (ap^2 + bp), find its common…
  24. If the sum of first n terms is (3n^2 + 5n), find its common difference.…
  25. Find an AP whose 4th term is 9 and the sum of its 6th and 13th terms is 40.…
Exercise 11d
  1. 2, 7, 12, 17, ... to 19 terms. Find the sum of each of the following APs:…
  2. 9, 7, 5, 3, ... to 14 terms. Find the sum of each of the following APs:…
  3. - 37, - 33, - 29, ... to 12 terms. Find the sum of each of the following APs:…
  4. (1/15), (1/12), (1/10),…….. to 11 terms. Find the sum of each of the following…
  5. 0.6, 1.7, 2.8, ... to 100 terms. Find the sum of each of the following APs:…
  6. 7+10 1/2 + 14 + l +84 Find the sum of each of the following arithmetic series:…
  7. 34 + 32 + 30 + … + 10. Find the sum of each of the following arithmetic…
  8. (-5) + (-8) + (-11) + … + (-230) Find the sum of each of the following…
  9. Find the sum of first n terms of an AP whose nth term is (5 - 6n). Hence, find…
  10. The sum of the first n terms of an AP is (3n^2 + 6n). Find the nth term and the…
  11. The sum of the first n terms of an AP is given by Sn = (3n^2 - n) . Find its…
  12. The sum of the first n terms of an AP is (5n^2/2 + 3n/2) . Find the nth term…
  13. The sum of the first n terms of an AP is (3n^2/2 + 5n/2) . Find its nth term…
  14. How many terms of the AP 21, 18, 15, ... must be added to get the sum 0?…
  15. How many terms of the AP 9, 17, 25, ... must be taken so that their sum is 636?…
  16. How many terms of the AP 63, 60, 57, 54, ... must be taken so that their sum…
  17. How many terms of the AP 20 , 19 1/3 , 18 2/3 ,... must be taken so that their…
  18. Find the sum of all odd numbers between 0 and 50.
  19. Find the sum of all natural numbers between 200 and 400 which are divisible by…
  20. Find the sum of first forty positive integers divisible by 6.
  21. Find the sum of the first 15 multiples of 8.
  22. Find the sum of all multiples of 9 lying between 300 and 700.
  23. Find the sum of all three - digit natural numbers which are divisible by 13.…
  24. Find the sum of first 100 even natural numbers which are divisible by 5.…
  25. Find the sum of the following. (1 - 1/n) + (1 - 2/n) + (1 - 3/n) + l up to n…
  26. In an AP, it is given that, S5 + S7 =167 and S10= 235, then find the AP, where…
  27. In an AP, the first term is 2, the last term is 29 and the sum of all the…
  28. In an AP, the first term is - 4, the last term is 29 and the sum of all its…
  29. The first and the last terms of an AP are 17 and 350 respectively. If the…
  30. The first and the last terms of an AP are 5 and 45 respectively. If the sum of…
  31. In an AP, the first term is 22, nth term is - 11 and sum of first n terms is…
  32. The 12th term of an AP is - 13 and the sum of its first four terms is 24. Find…
  33. The sum of the first 7 terms of an AP is 182. If its 4th and 17th terms are in…
  34. The sum of the first 9 terms of an AP is 81 and that of its first 20 terms is…
  35. The sum of the first 7 terms of an AP is 49 and the sum of its first 17 terms…
  36. Two APs have the same common difference. If the first terms of these APs be 3…
  37. The sum of first 10 terms of an AP is - 150 and the sum of its next 10 terms…
  38. The 13th term of an AP is 4 times its 3rd term. If its 5th term is 16, find…
  39. The 16th term of an AP is 5 times its 3rd term. If its 10th term is 41, find…
  40. An AP 5, 12, 19, ... has 50 terms. Find its last term. Hence, find the sum of…
  41. An AP 8, 10, 12, ... has 60 terms. Find its last term. Hence, find the sum of…
  42. The sum of the 4th and the 8th terms of an AP is 24 and the sum of its 6th and…
  43. The sum of first m terms of an AP is (4m^2 - m). If its nth term is 107, find…
  44. The sum of first q terms of an AP is (63q - 3q^2). If its pth term is - 60,…
  45. Find the number of terms of the AP - 12, - 9, - 6, ..., 21. If 1 is added to…
  46. Sum of the first 14 terms of an AP is 1505 and its first term is 10. Find its…
  47. Find the sum of first 51 terms of an AP whose second and third terms are 14…
  48. In a school, students decided to plant trees in and around the school to…
  49. In a potato race, a bucket is placed at the starting point, which is 5 m from…
  50. There are 25 trees at equal distances of 5 m in a line with a water tank, the…
  51. A sum of Rs. 700 is to be used to give seven cash prizes to students of a…
  52. A man saved Rs. 33000 in 10 months. In each month after the first, he saved…
  53. A man arranges to pay off a debt of Rs. 36000 by 40 monthly instalments which…
  54. A contract on construction job specifies a penalty for delay of completion…
Multiple Choice Questions (mcq)
  1. The common difference of the AP 1/p , 1-p/p , 1-2p/p , isA. p B. - p C. - 1 D. 1…
  2. The common difference of the AP 1/3 , 1-3b/3 , 1-6b/3 , isA. -1/3 B. -1/3 C. b D. - b…
  3. The next term of the AP root 7 , root 28 , root 63 , isA. root 112 B. root 84 C. root…
  4. If 4, x1, x2, x3, 28 are in AP then x3 = ?A. 19 B. 23 C. 22 D. cannot be determined…
  5. If the nth term of an AP is (2n + 1), then the sum of its first three terms isA. 6n + 3…
  6. The sum of first n terms of an AP is (3n^2 + 6n). The common difference of the AP isA.…
  7. The sum of first n terms of an AP is (5n - n^2). The nth term of the AP isA. (5 - 2n)…
  8. The sum of first n terms of an AP is (4n^2 + 2n). The nth term of this AP isA. (6n - 2)…
  9. The 7th term of an AP is - 1 and its 16th term is 17. The nth term of the AP isA. (3n +…
  10. The 5th term of an AP is - 3 and its common difference is - 4. The sum of its first 10…
  11. The 5th term of an AP is 20 and the sum of its 7th and 11th terms is 64. The common…
  12. The 13th term of an AP is 4 times its 3rd term. If its 5th term is 16 then the sum of…
  13. An AP 5, 12, 19, ... has 50 terms. Its last term isA. 343 B. 353 C. 348 D. 362…
  14. The sum of first 20 odd natural numbers isA. 100 B. 210 C. 400 D. 420…
  15. The sum of first 40 positive integers divisible by 6 isA. 2460 B. 3640 C. 4920 D. 4860…
  16. How many two - digit numbers are divisible by 3?A. 25 B. 30 C. 32 D. 36…
  17. How many three - digit numbers are divisible by 9?A. 86 B. 90 C. 96 D. 100…
  18. What is the common difference of an AP in which a18 - a14 = 32?A. 8 B. - 8 C. 4 D. - 4…
  19. If an denotes the nth term of the AP 3, 8, 13, 18, ... then what is the value of (a30…
  20. Which term of the AP 72, 63, 54, ... is 0?A. 8th B. 9th C. 10th D. 11th…
  21. Which term of the AP 25, 20, 15, ... is the first negative term?A. 10th B. 9th C. 8th…
  22. Which term of the AP 21, 42, 63, 84, ... is 210?A. 9th B. 10th C. 11th D. 12th…
  23. What is 20th term from the end of the AP 3, 8, 13, ..., 253?A. 163 B. 158 C. 153 D.…
  24. (5 + 13 + 21 + + 181) =?A. 2476 B. 2337 C. 2219 D. 2139
  25. The sum of first 16 terms of the AP 10, 6, 2, ... isA. 320 B. - 320 C. - 352 D. - 400…
  26. How many terms of the AP 3, 7, 11, 15, ... will make the sum 406?A. 10 B. 12 C. 14 D.…
  27. The 2nd term of an AP is 13 and its 5th term is 25. What is its 17th term?A. 69 B. 73…
  28. The 17th term of an AP exceeds its 10th term by 21. The common difference of the AP…
  29. The 8th term of an AP is 17 and its 14th term is 29. The common difference of the AP…
  30. The 7th term of an AP is 4 and its common difference is - 4. What is its first term?A.…

Exercise 11a
Question 1.

Show that each of the progressions given below is an AP. Find the first term, common difference and next term of each.

9, 15, 21, 27, ...


Answer:

Here, T2 - T1 = 15 - 9 = 6

T3 - T2 = 21 - 15 = 6


T4 - T3 = 27 - 21 = 6


Since the difference between each consecutive term is same, ∴ the progression is an AP.


So, first term = 9


Common difference = 15 - 9 = 6


Next term = T5 = T4 + d = 27 + 6 = 33



Question 2.

Show that each of the progressions given below is an AP. Find the first term, common difference and next term of each.

11, 6, 1, - 4, ...


Answer:

Here, T2 - T1 = 6 - 11 = - 5

T3 - T2 = 1 - 6 = - 5


T4 - T3 = - 4 - 1 = - 5


Since the difference between each consecutive term is same, ∴ the progression is an AP.


So, first term = 11


Common difference = 6 - 11 = - 5


Next term = T5 = T4 + d = - 4 + (-5) = - 9



Question 3.

Show that each of the progressions given below is an AP. Find the first term, common difference and next term of each.



Answer:

Here, T2 - T1 = (-5/6) - (-1) = 1/6

T3 - T2 = (-2/3) - (-5/6) = 1/6


T4 - T3 = (-1/2) - (-2/3) = 1/6


Since the difference between each consecutive term is same, ∴ the progression is an AP.


So, first term = - 1


Common difference = (-5/6) - (-1) = 1/6


Next term = T5 = T4 + d


= (-1/2) + (1/6)


= (-2/6)


= (-1/3)



Question 4.

Show that each of the progressions given below is an AP. Find the first term, common difference and next term of each.

√2, √8, √18, √32,…


Answer:

Here, T2 - T1 = √8 - √2

= 2√2 - √2


= √2


T3 - T2 = = √18 - √8


= 3√2 - 2√2


= √2


T4 - T3 = = √32 - √18


= 4√2 - 3√2


= √2


Since the difference between each consecutive term is same, ∴ the progression is an AP.


So, first term = √2


Common difference = √8 - √2 = √2


Next term = T5 = T4 + d


= √32 + √2


= 4√2 + √2


= 5√2


= √50



Question 5.

Show that each of the progressions given below is an AP. Find the first term, common difference and next term of each.

√20, √45, √80, √125,…


Answer:

Here, T2 - T1 = √45 - √20

= 3√5 - 2√5


= √5


T3 - T2 = = √80 - √45


= 4√5 - 3√5


= √5


T4 - T3 = = √125 - √80


= 5√5 - 4√5


= √5


Since the difference between each consecutive term is same, ∴ the progression is an AP.


So, first term = √20


Common difference = √45 - √20 = √5


Next term = T5 = T4 + d


= √125 + √5


= 5√5 + √5


= 6√5


= √180



Question 6.

Find:

the 20th term of the AP 9, 13, 17, 21, ... .


Answer:

Here, First term = a = 9

Common difference = d = 13 - 9 = 4


To find = 20th term, ∴ n = 20


Using the formula for finding nth term of an A.P.,


an = a + (n - 1) × d


∴ an = 9 + (20 - 1) × 4


⇒ an = 9 + 19 × 4 = 9 + 76 = 85


∴ 20th term of the given AP is 85.



Question 7.

Find:

the 35th term of the AP 20, 17, 14, 11, .


Answer:

Here, First term = a = 20

Common difference = d = 17 - 20 = - 3


To find = 35th term, ∴ n = 35


Using the formula for finding nth term of an A.P.,


an = a + (n - 1) × d


∴ an = 20 + (35 - 1) × (-3)


⇒ an = 20 + 34 × (-3) = 20 - 102 = - 82


∴ 20th term of the given AP is - 82.



Question 8.

Find:

the 18th term of the AP √2, √18, √50, √98….


Answer:

The given AP can be rewritten as √2, 3√2, 5√2, 7√2….

Here, First term = a = √2


Common difference = d = 3√2 - √2 = 2√2


To find = 18th term, ∴ n = 18


Using the formula for finding nth term of an A.P.,


an = a + (n - 1) × d


∴ an = √2 + (18 - 1) × 2√2


⇒ an = √2 + 17 × 2√2 = √2 + 34√2 = 35√2


∴ 18th term of the given AP is 35√2.



Question 9.

Find:

the 9th term of the AP


Answer:

Here, First term = a = 3/4

Common difference = d = 5/4 - 3/4 = 2/4


To find = 9th term, ∴ n = 9


Using the formula for finding nth term of an A.P.,


an = a + (n - 1) × d


∴ an = (3/4) + (9 - 1) × (2/4)


⇒ an = 3/4 + 8 × (2/4) = 3/4 + 16/4 = 19/4


∴ 9th term of the given AP is 19/4.



Question 10.

Find:

the 15th term of the AP - 40, - 15, 10, 35,…


Answer:

Here, First term = a = - 40

Common difference = d = - 15 - (-40) = 25


To find = 15th term, ∴ n = 15


Using the formula for finding nth term of an A.P.,


an = a + (n - 1) × d


∴ an = - 40 + (15 - 1) × (25)


⇒ an = - 40 + 14 × (25) = - 40 + 350 = 310


∴ 15th term of the given AP is 310.



Question 11.

Find the 37th term of the AP


Answer:

The given AP can be rewritten as 6, 31/4 ,19/2 , 45/4,…

Here, First term = a = 6


Common difference = d = (31/4) - 6 = 7/4


To find = 37th term, ∴ n = 37


Using the formula for finding nth term of an A.P.,


an = a + (n - 1) × d


∴ an = 6 + (37 - 1) × (7/4)


⇒ an = 6 + 36 × (7/4) = 6 + 63 = 69


∴ 37th term of the given AP is 69.



Question 12.

Find the 25th term of the AP


Answer:

Here, First term = a = 5

Common difference = d = 9/2 - 5 = - (1/2)


To find = 25th term, ∴ n = 25


Using the formula for finding nth term of an A.P.,


an = a + (n - 1) × d


∴ an = 5 + (25 - 1) × (-1/2)


⇒ an = 5 + 24 × (-1/2) = 5 - 12 = - 7


∴ 25th term of the given AP is - 7.



Question 13.

Find the nth term of each of the following AP’s:

5, 11, 17, 23, ...


Answer:

Here, First term = a = 5

Common difference = d = 11 - 5 = 6


To find = nth term


Using the formula for finding nth term of an A.P.,


an = a + (n - 1) × d


∴ an = 5 + (n - 1) × 6


⇒ an = 5 + 6n - 6 = 6n - 1


∴ nth term of the given AP is (6n - 1).



Question 14.

Find the nth term of each of the following AP’s:

16, 9, 2, - 5, ...


Answer:

Here, First term = a = 16

Common difference = d = 9 - 16 = - 7


To find = nth term


Using the formula for finding nth term of an A.P.,


an = a + (n - 1) × d


∴ an = 16 + (n - 1) × (-7)


⇒ an = 16 - 7n + 7 = 23 - 7n


∴ nth term of the given AP is (23 - 7n).



Question 15.

If the nth term of a progression is (4n - 10) show that it is an AP. Find its

(i) first term,

(ii) common difference, and

(iii) 16th term.


Answer:

nth term of the AP is (4n - 10).

For n = 1, we have a1 = 4 - 10 = - 6


For n = 2, we have a2 = 8 - 10 = - 2


For n = 3, we have a3 = 12 - 10 = 2


For n = 4, we have a4 = 16 - 10 = 6, and so on.


∴ a4 - a3 = a3 - a2 = a2 - a1 = 4 = constant.


∴ the given progression is an AP.


Hence, (i) Its first term = a = - 6


(ii) common difference = 4


(iii) To find :16th term


∴ a16 = a + (16 - 1)d


⇒ a16 = - 6 + 15 × 4 = 54


∴ 16th term of the given AP is 54.



Question 16.

How many terms are there in the AP 6, 10, 14, 18, ... , 174?


Answer:

In the given AP, the first term = a = 6

Common difference = d = 10 - 6 = 4


Last term = 174


To find: No. of terms in the AP.


Since, we know that


an = a + (n - 1) × d


∴ 174 = 6 + (n - 1) × 4


⇒ 174 - 6 = 4n - 4


⇒ 168 = 4n - 4


⇒ 168 + 4= 4n


⇒ 4n = 172


⇒ n = 172/4


⇒ n = 43


∴ Number of terms = 43.



Question 17.

How many terms are there in the AP 41, 38, 35, ..., 8?


Answer:

In the given AP, the first term = a =41

Common difference = d = 38 - 41 = - 3


Last term = 8


To find: No. of terms in the AP.


Since, we know that


an = a + (n - 1) × d


∴ 8 = 41 + (n - 1) × (-3)


⇒ 8 - 41 = - 3n + 3


⇒ - 33 = - 3n + 3


⇒ - 33 - 3 = - 3n


⇒ - 3n = - 36


⇒ n = 36/3


⇒ n = 12


∴ Number of terms = 12.



Question 18.

How many terms are there in the AP 18, 15, 13, ..., - 47?


Answer:

In the given AP, the first term = a = 18

Common difference = d = (31/2) - 18 = (-5/2)


Last term = - 47


To find: No. of terms in the AP.


Since, we know that


an = a + (n - 1) × d


∴ - 47 = 18 + (n - 1) × (-5/2)


⇒ - 47 - 18= (n - 1) × (-5/2)


⇒ - 65 = (n - 1) × (-5/2)


⇒ - 65 × (-2/5) = n - 1


⇒ n - 1 = 26


⇒ n = 26 + 1


⇒ n = 27


∴ Number of terms = 27.



Question 19.

Which term of the AP 3, 8, 13, 18, ... is 88?


Answer:

In the given AP, the first term = a = 3

Common difference = d = 8 - 3 = 5


To find: place of the term 88.


So, let an = 88


Since, we know that


an = a + (n - 1) × d


∴ 88 = 3 + (n - 1) × 5


⇒ 88 - 3 = 5n - 5


⇒ 85 = 5n - 5


⇒ 85 + 5 = 5n


⇒ 5n = 90


⇒ n = 90/5


⇒ n = 18


∴ 18th term of the AP is 88.



Question 20.

Which term of the AP 72, 68, 64, 60, ... is 0?


Answer:

In the given AP, the first term = a = 72

Common difference = d = 68 - 72 = - 4


To find: place of the term 0.


So, let an = 0


Since, we know that


an = a + (n - 1) × d


∴ 0 = 72 + (n - 1) × (-4)


⇒ 0 - 72 = - 4n + 4


⇒ - 72 - 4 = - 4n


⇒ - 76 = - 4n


⇒ n = 76/4


⇒ n = 19


∴ 19th term of the AP is 0.



Question 21.

Which term of the AP is 3?


Answer:

In the given AP, the first term = a = 5/6

Common difference = d = 1 - 5/6 = 1/6


To find: place of the term 3.


So, let an = 3


Since, we know that


an = a + (n - 1) × d


∴ 3 = (5/6) + (n - 1) × (1/6)


⇒ 3 - (5/6) = (n - 1) × (1/6)


⇒ 13/6 = (n - 1) × (1/6)


⇒ 13 = n - 1


⇒ n = 13 + 1


⇒ n = 14


∴ 14th term of the AP is 3.



Question 22.

Which term of the AP 21, 18, 15, ... is - 81?


Answer:

In the given AP, the first term = a = 21

Common difference = d = 18 - 21 = - 3


To find: place of the term - 81.


So, let an = - 81


Since, we know that


an = a + (n - 1) × d


∴ - 81 = 21 + (n - 1) × (-3)


⇒ - 81 - 21 = - 3n + 3


⇒ - 102 = - 3n + 3


⇒ - 102 - 3 = - 3n


⇒ - 3n = - 105


⇒ n = 105/3


⇒ n = 35


∴ 35th term of the AP is - 81.



Question 23.

Which term of the AP 3, 8, 13, 18, ... will be 55 more than its 20th term?


Answer:

In the given AP, the first term = a = 3

Common difference = d = 8 - 3 = 5


To find: place of the term which is 55 more than its 20th term.


So, we first find its 20th term.


Since, we know that


an = a + (n - 1) × d


∴ a20 = 3 + (20 - 1) × 5


⇒ a20 = 3 + 19 × 5


⇒ a20 = 3 + 95


⇒ a20 = 98


∴ 20th term of the AP is 98.


Now, 55 more than 20th term of the AP is 55 + 98 = 153.


So, to find: place of the term 153.


So, let an = 153


Since, we know that


an = a + (n - 1) × d


∴ 153 = 3 + (n - 1) × 5


⇒ 153 - 3 = 5n - 5


⇒ 150 = 5n - 5


⇒ 150 + 5 = 5n


⇒ 5n = 155


⇒ n = 155/5 = 31


∴ 31st term of the AP is the term which is 55 more than 20th term.



Question 24.

Which term of the AP 5, 15, 25, ... will be 130 more than its 31st term?


Answer:

In the given AP, the first term = a = 5

Common difference = d = 15 - 5 = 10


To find: place of the term which is 130 more than its 31st term.


So, we first find its 31st term.


Since, we know that


an = a + (n - 1) × d


∴ a31 = 5 + (31 - 1) × 10


⇒ a31 = 5 + 30 × 10


⇒ a31 = 5 + 300


⇒ a31 = 305.


∴ 31st term of the AP is 305.


Now, 130 more than 31st term of the AP is 130 + 305 = 435.


So, to find: place of the term 435.


So, let an = 435


Since, we know that


an = a + (n - 1) × d


∴ 435 = 5 + (n - 1) × 10


⇒ 435 - 5 = 10n - 10


⇒ 430 = 10n - 10


⇒ 430 + 10 = 10n


⇒ 10n = 440


⇒ n = 440/10 = 44


∴ 44th term of the AP is the term which is 130 more than 31st term.



Question 25.

If the 10th term of an AP is 52 and 17th term is 20 more than its 13th term, find the AP.


Answer:

Given: 10th term of the AP is 52.

17th term is 20 more than the 13th term.


Let the first term be a and the common difference be d.


Since,


an = a + (n - 1) × d


therefore for 10th term, we have,


52 = a + (10 - 1) × d


⇒ 52 = a + 9d ………… (1)


Now, 17th term is 20 more than the 13th term.


∴ a17 = 20 + a13


⇒ a + (17 - 1)d = 20 + a + (13 - 1)d


⇒ 16d = 20 + 12d


⇒ 4d = 20


⇒ d= 5


∴ from equation (1), we have,


52 = a + 9d


⇒ 52 = a + 9 × 5


⇒ 52 = a + 45


⇒ a = 52 - 45


⇒ a = 7


∴ AP is a, a + d, a + 2d, a + 3d,…


∴ AP is 7, 12, 17, 22….



Question 26.

Find the middle term of the AP 6, 13, 20, ..., 216.


Answer:

First term of the AP = 6

Common difference = d = 13 - 6 = 7


Last term = 216


Since


an = a + (n - 1) × d


∴ 216 = 6 + (n - 1) × 7


⇒ 216 - 6 = 7n - 7


⇒ 210 = 7n - 7


⇒ 210 + 7 = 7n


⇒ 7n = 217


⇒ n = 217/7 = 31


∴ Middle term is (31 + 1)/2 = 16th


So, a16 = a + (16 - 1) × d


∴ a16 = 6 + 15 × 7


⇒ a16 = 6 + 105 = 111


∴ Middle term of the AP is 111.



Question 27.

Find the middle term of the AP 10, 7, 4, ..., (-62).


Answer:

First term of the AP = 10

Common difference = d = 7 - 10 = - 3


Last term = - 62


Since


an = a + (n - 1) × d


∴ - 62 = 10 + (n - 1) × (-3)


⇒ - 62 - 10 = - 3n + 3


⇒ - 72 = - 3n + 3


⇒ - 72 - 3 = - 3n


⇒ 3n = 75


⇒ n = 75/3 = 25


∴ Middle term is (25 + 1)/2 = 13th


So, a13 = a + (13 - 1) × d


∴ a13 = 10 + 12 × (-3)


⇒ a13 = 10 - 36 = - 26


∴ Middle term of the AP is - 26.



Question 28.

Find the sum of two middle most terms of the AP


Answer:

First term of the AP = - (4/3)

Common difference = d = - 1 - (-4/3) = - 1 + (4/3) = 1/3


Last term = 13/3


Since


an = a + (n - 1) × d


∴ 13/3 = (-4/3) + (n - 1) × (1/3)


⇒ (13/3) + (4/3) = (n - 1) × (1/3)


⇒ 17/3 = (n - 1) × (1/3)


⇒ 17 = n - 1


⇒ n = 17 + 1


⇒ n = 18


∴ Two middle most terms of the AP are 18/2 and (18/2) + 1 terms, i.e. 9th and 10th terms.


So, a9 = a + (9 - 1) × d


∴ a9 = (-4/3) + [8 × (1/3)]


⇒ a9 = (-4/3) + (8/3) = 4/3


Also, a10 = a9 + d


= (4/3) + (1/3)


= 5/3


Now, a10 + a9 = (4/3) + (5/3)


= 9/3


= 3


∴ Sum of two middle most terms of the AP is 3.



Question 29.

Find the 8th term from the end of the AP 7, 10, 13, ..., 184.


Answer:

Here, First term = a = 7

Common difference = d = 10 - 7 = 3


Last term = l = 184


To find: 8th term from end.


So, nth term from end is given by:


an = l - (n - 1)d


∴ 8th term from end is:


a8 = 184 - (8 - 1) × 3


= 184 - 21


= 163



Question 30.

Find the 6th term from the end of the AP 17, 14, 11, ..., (-40).


Answer:

Here, First term = a = 17

Common difference = d = 14 - 17 = - 3


Last term = l = - 40


To find: 6th term from end.


So, nth term from end is given by:


an = l - (n - 1)d


∴ 6th term from end is:


a6 = - 40 - (6 - 1) × (-3)


= - 40 + 15


= - 25



Question 31.

Is 184 a term of the AP 3, 7, 11, 15, ...?


Answer:

Here, First term = a = 3

Common difference = d = 7 - 3 = 4


Now, to check: 184 is a term of the AP or not.


Since, nth term of an AP is given by:


an = a + (n - 1)d


If 184 is a term of the AP, then it must satisfy this equation.


So, let an = 184


∴ 184 = 3 + (n - 1) × 4


⇒ 184 - 3 = 4n - 4


⇒ 181 = 4n - 4


⇒ 181 + 4 = 4n


⇒ 4n = 185


⇒ n = 185/4 = 46.25


But this is not possible because n is the number of terms which can’t be a fraction.


Therefore, 184 is not a term of the given AP.



Question 32.

Is - 150 a term of the AP 11, 8, 5, 2, ...?


Answer:

Here, First term = a = 11

Common difference = d = 8 - 11 = - 3


Now, to check: - 150 is a term of the AP or not.


Since, nth term of an AP is given by:


an = a + (n - 1)d


If - 150 is a term of the AP, then it must satisfy this equation.


So, let an = - 150


∴ - 150 = 11 + (n - 1) × (-3)


⇒ - 150 - 11 = - 3n + 3


⇒ - 161 = - 3n + 3


⇒ - 161 - 3 = - 3n


⇒ 3n = 164


⇒ n = 164/3 = 54.66


But this is not possible because n is the number of terms which can’t be a fraction.


Therefore, - 150 is not a term of the given AP.



Question 33.

Which term of the AP 121, 117, 113, ... is its first negative term?


Answer:

Here, First term = a = 121

Common difference = d = 117 - 121 = - 4


Let nth term of the AP be its first negative term.


∴ an <0


Since, nth term of an AP is given by:


an = a + (n - 1)d


∴ a + (n - 1)d < 0


⇒ 121 + (n - 1) × (-4) < 0


⇒ - 4n + 125 < 0


⇒ - 4n < - 125


⇒ 4n > 125


⇒ n > 31.25


Since n is an integer, therefore n must be 32.


∴ 32nd term will be the first negative term of the AP.



Question 34.

Which term of the AP is its first negative term?


Answer:

Here, First term = a = 20

Common difference = d = (77/4) - 20 = (-3/4)


Let nth term of the AP be its first negative term.


∴ an <0


Since, nth term of an AP is given by:


an = a + (n - 1)d


∴ a + (n - 1)d < 0


⇒ 20 + (n - 1) × (-3/4) < 0


⇒ 80 + (n - 1) × (-3) < 0 (multiplying both sides by 4)


⇒ 80 - 3n + 3 < 0


⇒ - 3n < - 83


⇒ 3n > 83


⇒ n > 27.66


Since n is an integer, therefore n must be 28.


∴ 28th term will be the first negative term of the AP.



Question 35.

The 7th term of an AP is - 4 and its 13th term is - 16. Find the AP.


Answer:

Let a be the first term and d be the common difference.

Given: a7 = - 4


a13 = - 16


Now, Consider a7 = - 4


⇒ a + (7 - 1)d = - 4


⇒ a + 6d = - 4 ……….(1)


Consider a13 = - 16


⇒ a + (13 - 1)d = - 16


⇒ a + 12d = - 16 ……….(2)


Now, subtracting equation (1) from (2), we get,


6d = - 12


⇒ d = - 2


∴ from equation (1), we get,


a = - 4 - 6d


⇒ a = - 4 - 6 × (-2)


⇒ a = - 4 + 12


⇒ a = 8


Thus the AP is a, a + d, a + 2d, a + 3d, a + 4d,….


Therefore the AP is 8, 6, 4, 2, 0,….



Question 36.

The 4th term of an AP is zero. Prove that its 25th term is triple its 11th term.


Answer:

Let a be the first term and d be the common difference.

Given: a4 = 0


To prove: a25 = 3 × a11


Now, Consider a4 = 0


⇒ a + (4 - 1)d = 0


⇒ a + 3d = 0


⇒ a = - 3d ………. (1)


Consider a25 = a + (25 - 1)d


⇒ a25 = - 3d + 24d (from equation (1))


⇒ a25 = 21d ………. (2)


Now, consider a11 = a + (11 - 1)d


⇒ a11 = - 3d + 10d (from equation (1))


⇒ a11 = 7d ……….(3)


From equation (2) and (3), we get,


a25 = 3 × a11


Hence, proved.



Question 37.

The 8th term of an AP is zero. Prove that its 38th term is triple its 18th term.


Answer:

Let a be the first term and d be the common difference.

Given: a8 = 0


To prove: a38 = 3 × a18


Now, Consider a8 = 0


⇒ a + (8 - 1)d = 0


⇒ a + 7d = 0


⇒ a = - 7d ……. (1)


Consider a38 = a + (38 - 1)d


⇒ a38 = - 7d + 37d (from equation (1))


⇒ a38 = 30d ………. (2)


Now, consider a18 = a + (18 - 1)d


⇒ a18 = - 7d + 17d (from equation (1))


⇒ a18 = 10d ……….(3)


From equation (2) and (3), we get,


a38 = 3 × a18


Hence, proved.



Question 38.

The 4th term of an AP is 11. The sum of the 5th and 7th terms of this AP is 34. Find its common difference.


Answer:

Let a be the first term and d be the common difference.

Given: a4 = 11


a5 + a7 = 34


To find: common difference = d


Now, Consider a4 = 11


⇒ a + (4 - 1)d = 11


⇒ a + 3d = 11 ………….(1)


Consider a5 + a7 = 34


⇒ a + (5 - 1)d + a + (7 - 1)d = 34


⇒ 2a + 10d = 34


⇒ a + 5d = 17 ……..(2)


Subtracting equation (1) from equation (2), we get,


2d = 6


⇒ d = 3


∴ Common difference = d = 3



Question 39.

The 9th term of an AP is - 32 and the sum of its 11th and 13th terms is - 94. Find the common difference of the AP.


Answer:

Let a be the first term and d be the common difference.

Given: a9 = - 32


a11 + a13 = - 94


To find: common difference = d


Now, Consider a9 = - 32


⇒ a + (9 - 1)d = - 32


⇒ a + 8d = - 32 ……….(1)


Consider a11 + a13 = - 94


⇒ a + (11 - 1)d + a + (13 - 1)d = - 94


⇒ 2a + 22d = - 94


⇒ a + 11d = - 47 ……..(2)


Subtracting equation (1) from equation (2), we get,


3d = - 15


⇒ d = - 5


∴ Common difference = d = - 5


Question 40.

Determine the nth term of the AP whose 7th term is - 1 and 16th term is 17.


Answer:

Let a be the first term and d be the common difference.

Given: a7 = - 1


a16 = 17


Now, Consider a7 = - 1


⇒ a + (7 - 1)d = - 1


⇒ a + 6d = - 1 ……….(1)


Consider a16 = 17


⇒ a + (16 - 1)d = 17


⇒ a + 15d = 17 ……….(2)


Now, subtracting equation (1) from (2), we get,


9d = 18


⇒ d = 2


∴ from equation (1), we get,


a = - 1 - 6d


⇒ a = - 1 - 6 × (2)


⇒ a = - 1 - 12


⇒ a = - 13


Now, the nth term of the AP is given by:


an = a + (n - 1)d


∴ an = - 13 + (n - 1) × 2


⇒ an = 2n - 15


∴ nth term of the AP is (2n - 15)



Question 41.

If 4 times the 4th term of an AP is equal to 18 times its 18th term then find its 22nd term.


Answer:

Given: 4 × a4 = 18 × a18

To find : a22


Consider 4 × a4 = 18 × a18


⇒ 4 [a + (4 - 1)d] = 18 [a + (18 - 1)d]


⇒ 4a + 12d = 18a + 306d


⇒ - 14 a = 294 d


⇒ a = - 21d ………………(1)


Now, a22 = a + (22 - 1)d


⇒ a22 = a + 21d


⇒ a22 = - 21d + 21d (from equation 1)


⇒ a22 = 0


∴ a22 = 0



Question 42.

If 10 times the 10th term of an AP is equal to 15 times the 15th term, show that its 25th term is zero.


Answer:

Given: 10 × a10 = 15 × a15

To show : a25 = 0


Consider 10 × a10 = 15 × a15


⇒ 10 [a + (10 - 1)d] = 15 [a + (15 - 1)d]


⇒ 10a + 90d = 15a + 210d


⇒ - 5 a = 120 d


⇒ a = - 24d ………………(1)


Now, a25 = a + (25 - 1)d


⇒ a25 = a + 24d


⇒ a25 = - 24d + 24d (from equation 1)


⇒ a25 = 0


Hence, proved.



Question 43.

Find the common difference of an AP whose first term is 5 and the sum of its first four terms is half the sum of the next four terms.


Answer:

Let a be the first term and d be the common difference of the AP.

Given: a = 5


Sum of first four terms = 1/2(sum of next four terms)


⇒ a + (a + d) + (a + 2d) + (a + 3d) = 1/2 ((a + 4d) + (a + 5d) + (a + 6d) +


(a + 7d))


⇒ 4a + 6d = 1/2(4a + 22d)


⇒ 4a + 6d = 2a + 11d


⇒ 2a = 5d


⇒ d = 2a/5


As a = 5, therefore,


d = 10/5 = 2


Thus, Common difference = d = 2



Question 44.

The sum of the 2nd and the 7th term of an AP is 30. If its 15th term is 1 less than twice its 8th term, find the AP.


Answer:

Let a be the first term and d be the common difference of the AP.

Given: a2 + a7 = 30


Also, a15 = 2a8 - 1


Consider a2 + a7 = 30


⇒ (a + d) + (a + 6d) = 30


⇒ 2a + 7d = 30 ……………….. (1)


Consider a15 = 2a8 - 1


⇒ a + 14d = 2(a + 7d) - 1


⇒ a + 14d = 2a + 14d - 1


⇒ a = 1


∴ First term = a = 1


Thus, from equation (1), we get,


7d = 30 - 2a


⇒ 7d = 30 - 2


⇒ 7d = 28


⇒ d = 4


Thus, the AP is a, a + d, a + 2d, a + 3d,…


Therefore, the AP is 1, 5, 9, 13, 17,…



Question 45.

For what value of n, the nth terms of the arithmetic progressions 63, 65, 67, ... and 3, 10, 17, ... are equal?


Answer:

Let a1 and d1 be the first term and common difference of the AP 63, 65, 67, 69,….

Let a2 and d2 be the first term and common difference of the AP 3, 10, 17,….


∴ a1 = 63, d1 = 2


a2 = 3, d2 = 7


Let an be the nth term of the first AP and bn be the nth term of the second AP.


So, an = a1 + (n - 1)d1


⇒ an = 63 + (n - 1)2


⇒ an = 61 + 2n


and, bn = a2 + (n - 1)d2


⇒ bn = 3 + (n - 1)7


⇒ bn = - 4 + 7n


Since for nth terms of both the AP’s to be same, an = bn


⇒ 61 + 2n = - 4 + 7n


⇒ 61 + 4= 7n - 2n


⇒ 65 = 5n


⇒ n = 13


Therefore, 13th term of both the AP’s will be same.



Question 46.

The 17th term of AP is 5 more than twice its 8th term. If the 11th term of the AP is 43, find its nth term.


Answer:

Let a and d be the first term and common difference of the AP Given: a17 = 2 × a8 + 5

a11 = 43


To find: nth term = an


Consider a11 = 43


⇒ a + (11 - 1)d = 43


⇒ a + 10d = 43 ………….. (1)


Consider a17 = 2 × a8 + 5


⇒ a + (17 - 1)d = 2[a + (8 - 1)d] + 5


⇒ a + 16d = 2a + 14d + 5


⇒ - a + 2d = 5 ……………….(2)


Adding equation (1) and equation (2), we get


12d = 48


⇒ d = 4


∴ from equation (1), we get,


a = 43 - 10d


= 43 - 40


= 3


Now, nth term is given by:


an = a + (n - 1)d


⇒ an = 3 + (n - 1)4


⇒ an = 4n - 1


Therefore, nth term is given by (4n - 1).



Question 47.

The 24th term of an AP is twice its 10th term. Show that its 72nd term is 4 times its 15th term.


Answer:

Let a be the first term and d be the common difference.

Given: a24 = 2(a10)


To prove: a72 = 4 × a15


Now, Consider a24 = 2a10


⇒ a + 23d = 2[a + 9d]


⇒ a + 23d = 2a + 18d


⇒ a = 5d …………………. (1)


Consider a72 = a + (72 - 1)d


⇒ a72 = 5d + 71d (from equation (1))


⇒ a72 = 76d ………………. (2)


Now, consider a15 = a + (15 - 1)d


⇒ a15 =5d + 14d (from equation (1))


⇒ a18 = 19d ………………….(3)


From equation (2) and (3), we get,


a72 = 4 × a15


Hence, proved.



Question 48.

The 19th term of an AP is equal to 3 times its 6th term. If its 9th term is 19, find the AP.


Answer:

Let a be the first term and d be the common difference.

Given: a9 = 19


a19 = 3 a6


Now, Consider a9 = 19


⇒ a + (9 - 1)d = 19


⇒ a + 8d = 19 ………………….(1)


Consider a19 = 3 a6


⇒ a + 18d = 3(a + 5d)


⇒ a + 18d = 3a + 15d


⇒ 2a - 3d = 0 ………………….(2)


Now, subtracting twice of equation (1) from (2), we get,


- 19d = - 38


⇒ d = 2


∴ from equation (1), we get,


a = 19 - 8d


⇒ a = 19 - 8 × 2


⇒ a = 19 - 16


⇒ a = 3


Thus the AP is a, a + d, a + 2d, a + 3d, a + 4d,….


Therefore the AP is 3, 5, 7, 9….



Question 49.

If the pth term of an AP is q and its qth term is p then show that its (p + q)th term is zero.


Answer:

Let a be the first term and d be common difference.

Given: ap = q

aq = p

To show: a(p + q) = 0


We know, nth term of an AP is
an = a + (n - 1)d
where, a is first term and d is common difference
Consider ap = q

⇒ a + (p - 1)d = q (1)


Consider aq = p


⇒ a + (q - 1)d = p (2)


Now, subtracting equation (2) from equation (1), we get

(p - q)d = (q - p)

⇒ d = - 1


∴ From equation (1), we get,

a - p + 1 = q

⇒ p + q = a + 1 ……………………….(3)


Consider a(p + q) = a + (p + q - 1)d

= a + (p + q - 1)(-1)

= a + (a + 1 - 1)(-1)

(putting the value of p + q from equation 3)

= a + (-a)

= 0

∴ a(p + q) = 0

Hence, proved.


Question 50.

The first and last terms of an AP are a and 1 respectively. Show that the sum of the nth term from the beginning and the nth term from the end is (a + 1).


Answer:

Let d be the common difference of the AP.

First term = a


Last term = l = 1


nth term from beginning of an AP is given by:


an = a + (n - 1)d …………………………….(1)


nth term from the end of an AP is given by:


Tn = l - (n - 1)d


= 1 - (n - 1)d …………………………….(2)


Sum of the nth term from the beginning and end is given by:


an + Tn = a + (n - 1)d + 1 - (n - 1)d


= a + 1


Hence, proved.



Question 51.

How many two - digit numbers are divisible by 6?


Answer:

The two digit numbers divisible by 6 are 12, 18, 24, 30,…96.

This forms an AP with first term a = 12


and common difference = d = 6


Last term is 96.


Now, number of terms in this AP are given as:


96 = a + (n - 1)d


⇒ 96 = 12 + (n - 1)6


⇒ 96 - 12 = 6n - 6


⇒ 84 + 6 = 6n


⇒ 90 = 6n


⇒ n = 15


There are 15 two - digit numbers that are divisible by 6.



Question 52.

How many two - digit numbers are divisible by 3?


Answer:

The two digit numbers divisible by 3 are 12, 15, 18, 21, …., 99.

This forms an AP with first term a = 12


and common difference = d = 3


Last term is 99.


Now, number of terms in this AP are given as:


99 = a + (n - 1)d


⇒ 99 = 12 + (n - 1)3


⇒ 99 - 12 = 3n - 3


⇒ 87 + 3 = 3n


⇒ 90 = 3n


⇒ n = 30


There are 30 two - digit numbers that are divisible by 3.



Question 53.

How many three - digit numbers are divisible by 9?


Answer:

The three digit numbers divisible by 9 are 108, 117, 126, …., 999.

This forms an AP with first term a = 108


and common difference = d = 9


Last term is 999.


Now, number of terms in this AP are given as:


999 = a + (n - 1)d


⇒ 999 = 108 + (n - 1)9


⇒ 999 - 108 = 9n - 9


⇒ 891 + 9 = 9n


⇒ 900 = 9n


⇒ n = 100


There are 100 three - digit numbers that are divisible by 9.



Question 54.

How many numbers are there between 101 and 999, which are divisible by both 2 and 5?


Answer:

The numbers between 101 and 999 that are divisible by both 2 and 5 are 110, 120, 130,…, 990.

This forms an AP with first term a = 110


and common difference = d = 10


Last term is 990.


Now, number of terms in this AP are given as:


990 = a + (n - 1)d


⇒ 990 = 110 + (n - 1)10


⇒ 990 - 110 = 10n - 10


⇒ 880 + 10 = 10n


⇒ 890 = 10n


⇒ n = 89


There are 89 numbers between 101 and 999 that are divisible by both 2 and 5.



Question 55.

In a flower bed, there are 43 rose plants in the first row, 41 in the second, 39 in the third, and so on. There are 11 rose plants in the last row. How many rows are there in the flower bed?


Answer:

The no of rose plants in each row can be arranged in the form of an AP as 43, 41, 39, …, 11.

Here, First term = a = 43


Common difference = d = 41 - 43 = - 2


No of terms in the AP = No of rows in the flower bed.


∴ 11 = a + (n - 1)d


⇒ 11 = 43 + (n - 1)(-2)


⇒ 11 - 43 = - 2n + 2


⇒ 11 - 43 - 2 = - 2n


⇒ 2n = 34


⇒ n = 17


∴ No of rows in the flower bed = 17



Question 56.

A sum of Rs. 2800 is to be used to award four prizes. If each prize after the first is Rs. 200 less than the preceding prize, find the value of each of the prizes.


Answer:

Let the first prize be Rs. x. Thus each succeeding prize is Rs. 200 less than the preceding prize.

∴ Second prize is Rs. (x - 200)


Third prize is Rs. (x - 400)


Fourth prize is Rs. (x - 600)


This forms an AP as x, x - 200, x - 400, x - 600.


Since, Total sum of prize amount = 2800.


x + (x - 200) + (x - 400) + (x - 600) = 2800


⇒ 4x - 1200 = 2800


⇒ 4x = 2800 + 1200


⇒ 4x = 4000


x = 1000


Thus, the first, second, third and fourth prizes are as Rs. 1000, Rs. 800, Rs. 600, Rs. 400.




Exercise 11b
Question 1.

Determine k so that (3k - 2), (4k - 6) and (k + 2) are three consecutive terms of an AP.


Answer:

If three terms are in AP, the difference between the terms should be equal, i.e. if a, b and c are in AP then,
b - a = c - b

Since, the terms are in an AP, therefore

(4k - 6) - (3k - 2) = (k + 2) - (4k - 6)


⇒ k - 4 = - 3k + 8


⇒ 4k = 12


⇒ k = 3


∴ k = 3


Question 2.

Find the value of x for which the numbers (5x + 2), (4x - 1) and (x + 2) are in AP.


Answer:

Given: The numbers (5x + 2), (4x - 1) and (x + 2) are in AP.

To find: The value of x.

Solution:

Let a1 = (5x + 2)

a2 = 4x - 1)

a3 = (x + 2)
Since, the terms are in an AP, therefore common difference is same.

⇒ a2 - a1 = a3 - a2

⇒(4x - 1) - (5x + 2) = (x + 2) - (4x - 1)

⇒ 4x - 1 - 5x - 2 = x + 2 - 4x +1

⇒ - x - 3 = - 3x + 3

⇒ -x + 3x = 3 + 3

⇒ 2x = 6

⇒ x = 3

∴ x = 3


Question 3.

If (3y - 1), (3y + 5) and (5y + 1) are three consecutive terms of an AP then find the value of y.


Answer:

Since, the terms are in an AP, therefore

(3y + 5) - (3y - 1) = (5y + 1) - (3y + 5)


⇒ 6 = 2y - 4


⇒ 2y = 10


⇒ y = 5


∴ y = 5



Question 4.

Find the value of x for which (x + 2), 2x, (2x + 3) are three consecutive terms of an AP.


Answer:

Given: (x + 2), 2x, (2x + 3) are three consecutive terms of an AP.

To find:
the value of x

Solution:

Let a1 = x + 2

a2 = 2x

a3 = 2x + 3

As, a1, a2 and a3 are in AP, common difference will be equal

⇒ a2 – a1 = a3 – a2

⇒ (2x) - (x + 2) = (2x + 3) - (2x)

⇒ 2x - x - 2= 2x + 3 - 2x

⇒ x - 2 = 3

⇒ x = 5


Question 5.

Show that (a - b)2, (a2 + b2) and (a + b)2 are in AP.


Answer:

Consider (a2 + b2) - (a - b)2

= (a2 + b2) - (a2 + b2 - 2ab)


= 2ab


Consider (a + b)2 - (a2 + b2)


= (a2 + b2 + 2ab) - (a2 + b2)


= 2ab


Since, the difference between consecutive terms is constant, therefore the terms are in AP.



Question 6.

Find three numbers in AP whose sum is 15 and product is 80.


Answer:

Let the numbers be (a - d), a, (a + d).

Now, sum of the numbers = 15


∴ (a - d) + a + (a + d) = 15


⇒ 3a = 15


⇒ a = 5


Now, product of the numbers = 80


⇒ (a - d) × a × (a + d) = 80


⇒ a3 - ad2 = 80


Put the value of a, we get,


125 - 5 d2 = 80


⇒ 5 d2 = 125 - 80 = 45


d2 = 9


d = � 3


∴ If d = 3, then the numbers are 2, 5, 8.


If d = - 3, then the numbers are 8, 5, 2.



Question 7.

The sum of three numbers in AP is 3 and their product is - 35. Find the numbers.


Answer:

Let the numbers be (a - d), a, (a + d).

Now, sum of the numbers = 15


∴ (a - d) + a + (a + d) = 3


⇒ 3a = 3


⇒ a = 1


Now, product of the numbers = - 35


⇒ (a - d) × a × (a + d) = - 35


⇒ a3 - ad2 = - 35


Put the value of a, we get,


1 - d2 = - 35


⇒ d2 = 35 + 1 = 36


d2 = 36


d = 6


∴ If d = 6, then the numbers are - 5, 1, 7.


If d = - 6, then the numbers are 7, 1, - 5.



Question 8.

Divide 24 in three parts such that they are in AP and their product is 440.


Answer:

Let 24 be divided in numbers which are in AP as (a - d), a, (a + d).

Now, sum of the numbers = 24


∴ (a - d) + a + (a + d) = 24


⇒ 3a = 24


⇒ a = 8


Now, product of the numbers = 440


⇒ (a - d) × a × (a + d) = 440


⇒ a3 - ad2 = 440


Put the value of a, we get,


512 - 8d2 = 440


⇒ 8d2 = 512 - 440 = 72


d2 = 9


d = � 3


∴ If d = 3, then the numbers are 5, 8, 11.


If d = - 3, then the numbers are 11, 8, 5.



Question 9.

The sum of three consecutive terms of an AP is 21 and the sum of the squares of these terms is 165. Find these terms.


Answer:

Let the numbers be (a - d), a, (a + d).

Now, sum of the numbers = 21


∴ (a - d) + a + (a + d) = 21


⇒ 3a = 21


⇒ a = 7


Now, sum of the squares of the terms = 165


⇒ (a - d)2 + a2 + (a + d)2 = 165


⇒ a2 + d2 - 2ad + a2 + a2 + d2 + 2ad = 165


⇒ 3a2 + 2d2 + a = 165


Put the value of a = 7, we get,


3(49) + 2d2 = 165


⇒ 2d2 = 165 - 147


⇒ 2d2 = 18


⇒ d2 = 9


⇒ d = 3


∴ If d = 3, then the numbers are 4, 7, 10.


If d = - 3, then the numbers are 10, 7, 4.



Question 10.

The angles of a quadrilateral are in AP whose common difference is 10°. Find the angles.


Answer:

Let these angles be x°, (x + 10)°, (x + 20)° and (x + 30)°.

Since, Sum of all angles of a quadrilateral = 360°.


⇒ x° + (x + 10)° + (x + 20)° + (x + 30)° = 360°


⇒ 4x + 60° = 360°


⇒ 4x = 300°


⇒ x = 75°


∴ the angles will be 75°, 85°, 95°, 105°.



Question 11.

Find four numbers in AP whose sum is 28 and the sum of whose squares is 216.


Answer:

Let the numbers be (a - 3d), (a - d), (a + d), (a + 3d).

Now, sum of the numbers = 28


∴ (a - 3d) + (a - d) + (a + d) + (a + 3d) = 28


⇒ 4a = 28


⇒ a = 7


Now, sum of the squares of the terms = 216


⇒ (a - 3d)2 + (a - d)2 + (a + d)2 + (a + 3d)2= 216


⇒ a2 + 9d2 - 6ad + a2 + d2 - 2ad + a2 + d2 + 2ad + a2 + 9d2 + 6ad = 216


⇒ 4a2 + 20d2 = 216


Put the value of a = 7, we get,


4(49) + 20d2 = 216


⇒ 20d2 = 216 - 196


⇒ 20d2 = 20


⇒ d2 = 1


⇒ d = 1


∴ If d = 1, then the numbers are 4, 6, 8, 10.


If d = - 1, then the numbers are 10, 8, 6, 4.



Question 12.

Divide 32 into four parts which are the four terms of an AP such that the product of the first and the fourth terms is to the product of the second and the third terms as 7 : 15.


Answer:

Let 32 be divided into parts as (a - 3d), (a - d), (a + d) and (a + 3d).

Now (a - 3d) + (a - d) + (a + d) + (a + 3d) = 32

⇒ 4a = 32

⇒ a = 8

Now, we are given that product of the first and the fourth terms is to the product of the second and the third terms as 7 : 15.

i.e. [(a - 3d) × (a + 3 d)] : [(a - d) × (a + d)] = 7 : 15

=

⇒ 15[(a - 3d) × (a + 3 d)] = 7[(a - d) × (a + d)]

⇒ 15[a2 - 9d2] = 7 [a2 - d2]

⇒ 15a2 - 135d2 = 7a2 - 7d2

⇒ 8a2 - 128d2 = 0

⇒ 8a2 = 128d2

Putting the value of a, we get,

512 = 128 d2

⇒ d2 = 4

⇒ d = ±2

∴ If d = 2, then the numbers are 2, 6, 10, 14.

If d = - 2, then the numbers are 14, 10, 6, 2.


Question 13.

The sum of first three terms of an AP is 48. If the product of first and second terms exceeds 4 times the third term by 12. Find the AP.


Answer:

Let the numbers be (a - d), a, (a + d).

Now, sum of the numbers = 48

∴ (a - d) + a + (a + d) = 48

⇒ 3a = 48

⇒ a = 16


Now, we are given that,

Product of first and second terms exceeds 4 times the third term by 12.

⇒ (a - d) × a = 4(a + d) + 12

⇒ a2 - ad = 4a + 4d + 12


On putting the value of a in the above equation, we get,

256 - 16d = 64 + 4d + 12

⇒ 20 d = 180

⇒ d = 9


∴ The numbers are a - d, a, a + d


i.e. the numbers are 7, 16, 25.



Exercise 11c
Question 1.

The first three terms of an AP are respectively (3y – 1), (3y + 5) and (5y + 1), find the value of y.


Answer:

Since, the terms are in an AP, therefore

(3y + 5) - (3y - 1) = (5y + 1) - (3y + 5)


⇒ 6 = 2y - 4


⇒ 2y = 10


⇒ y = 5


∴ y = 5



Question 2.

If k, (2k – 1) and (2k + 1) are the three successive terms of an AP, find the value of k.


Answer:

Since, the terms are in an AP, therefore

(2k - 1) - k = (2k + 1) - (2k - 1)


⇒ k - 1 = 2


⇒ k = 3


∴ k = 3



Question 3.

If 18, a, (b – 3) are in AP, then find the value of (2a – b).


Answer:

Since, the terms are in an AP, therefore

a - 18 = (b - 3) - a


⇒ 2a - b = - 3 + 18


⇒ 2a - b = 15


∴ 2a - b = 15



Question 4.

If the numbers a, 9, b, 25 form an AP, find a and b.


Answer:

Since, the terms are in an AP, therefore

9 - a = b - 9 = 25 - b


Consider b - 9 = 25 - b


⇒ 2b = 34


⇒ b = 17


Now, consider the first equality,


9 - a = b - 9


⇒ a = 18 - b


⇒ a = 18 - 17


⇒ a = 1


∴ a = 1, b = 17



Question 5.

If the numbers (2n – 1), (3n + 2) and (6n – 1) are in AP, find the value of n and the numbers.


Answer:

Since, the terms are in an AP, therefore

(3n + 2) - (2n - 1) = (6n - 1) - (3n + 2)


⇒ n + 3 = 3n - 3


⇒ 2n = 6


⇒ n = 3


∴ n= 3, and hence the numbers are 5, 11, 17.



Question 6.

How many three - digit natural numbers are divisible by 7?


Answer:

The three digit numbers divisible by 7 are 105, 112, 119, …., 994.

This forms an AP with first term a = 105


and common difference = d = 7


Last term is 994.


Now, number of terms in this AP are given as:


994 = a + (n - 1)d


⇒ 994 = 105 + (n - 1)7


⇒ 994 - 105 = 7n - 7


⇒ 889 + 7 = 7n


⇒ 896 = 7n


⇒ n = 128


Therefore 994 is the 128th term in the AP.


∴ There are 128 three - digit natural numbers that are divisible by 7.



Question 7.

How many three - digit natural numbers are divisible by 9?


Answer:

The three digit natural numbers divisible by 9 are 108, 117, 126, …., 999.

This forms an AP with first term a = 108


and common difference = d = 9


Last term is 999.


Now, number of terms in this AP are given as:


999 = a + (n - 1)d


⇒ 999 = 108 + (n - 1)9


⇒ 999 - 108 = 9n - 9


⇒ 891 + 9 = 9n


⇒ 900 = 9n


⇒ n = 100


Therefore 999 is the 100th term in the AP.


∴ There are 100 three - digit natural numbers that are divisible by 9.



Question 8.

If the sum of first m terms of an AP is (2m2 + 3m) then what is its second term?


Answer:

Let Sn denotes the sum of first n terms of an AP.

Sum of first m terms = Sm = 2m2 + 3m


Then nth term is given by: an = Sn - Sn - 1


We need to find the 2nd term, so put n = 2, we get


a2 = S2 - S1


= (2(2)2 + 3(2)) - (2(1)2 + 3(1))


= 14 - 5


= 9


∴ the second term of the AP is 9.



Question 9.

What is the sum of first n terms of the AP a, 3a, 5a, ....


Answer:

Here, first term = a

Common difference = 3a - a = 2a


Now, Sum of first n terms of an AP is given by:


Sn = [2a + (n - 1)d]


where a is the first term and d is the common difference.


∴ Sum of first n terms of given AP is given by:


Sn = [2a + (n - 1)2a]


= [2a + 2an - 2a]


= [2an]


= n2a



Question 10.

What is the 5th term from the end of the AP 2, 7, 12, ..., 47?


Answer:

Here, First term = a = 2

Common difference = d = 7 - 2 = 5


Last term = l = 47


To find:5th term from end.


So, nth term from end is given by:


an = l - (n - 1)d


∴ 5th term from end is:


a5 = 47 - (5 - 1) × 5


= 47 - 20


= 27


∴ 5th term from the end is 27.



Question 11.

If an denotes the nth term of the AP 2, 7, 12, 17, ..., find the value of (a30 – a20).


Answer:

Here, First term = a = 2

Common difference = d = 7 - 2 = 5


To find: a30 – a20


So, nth term is given by:


an = a + (n - 1)d


∴ 30th term is:


a30 = 2 + (30 - 1) × 5


= 2 + 145


= 147


Now, 20th term is:


a20 = 2 + (20 - 1) × 5


= 2 + 95


= 97


Now, (a30 – a20) = 147 - 97


= 50


∴ (a30 – a20) = 50



Question 12.

The nth term of an AP is (3n + 5). Find its common difference.


Answer:

nth term of an AP = an = 3n + 5

Common difference (= d) of an AP is the difference between a term and its preceding term.


∴ d = an - an - 1


= (3n + 5) - (3(n - 1) + 5)


= 3n + 5 - 3n + 3 - 5


= 3


∴ Common difference = 3



Question 13.

The nth term of an AP is (7 – 4n). Find its common difference.


Answer:

nth term of an AP = an = 7 - 4n

Common difference (= d) of an AP is the difference between a term and its preceding term.


∴ d = an - an - 1


= (7 - 4n) - (7 - 4(n - 1))


= 7 - 4n - 7 + 4n - 4


= - 4


∴ Common difference = - 4.



Question 14.

Write the next term of the AP


Answer:

Here, first term = √8

Common difference = √18 - √8 = √2


Next term = T4 = T3 + d


= √32 + √2


= 4√2 + √2


= 5√2


= √50



Question 15.

Write the next term of the AP


Answer:

Here, first term = √2

Common difference = √8 - √2 = 2√2 - √2 = √2


Next term = T4 = T3 + d


= √18 + √2


= 3√2 + √2


= 4√2


= √32


Question 16.

Which term of the AP 21, 18, 15, ... is zero?


Answer:

Here first term = 21

Common difference = 18 - 21 = - 3


Let an be the term which is zero.


∴ an = 0


⇒ a + (n - 1)d = 0


⇒ 21 + (n - 1)(-3) = 0


⇒ 21 - 3n + 3 = 0


⇒ 3n = 24


⇒ n = 8


∴ 8th term of the given AP will be zero.



Question 17.

Find the sum of first n natural numbers.


Answer:

First n natural numbers are 1, 2, 3,…, n.

To find: sum of these n natural numbers.


The natural numbers forms an AP with first term 1 and common difference 1.


Now, Sum of first n terms of an AP is given by:


Sn = [2a + (n - 1)d]


where a is the first term and d is the common difference.


∴ Sum of first n natural numbers is given by:


Sn = [2(1) + (n - 1)(1)]


= [2 + n - 1]


= [n + 1]


∴ Sum of first n natural numbers is n(n + 1)/2.



Question 18.

Find the sum of first n even natural numbers.


Answer:

First n even natural numbers are 2, 4, 6,…, 2n.

To find: sum of these n even natural numbers.


The even natural numbers forms an AP with first term 2 and common difference 2.


Now, Sum of first n terms of an AP is given by:


Sn = [2a + (n - 1)d]


where a is the first term and d is the common difference.


∴ Sum of first n natural numbers is given by:


Sn = [2(2) + (n - 1)(2)]


= [4 + 2n - 2]


= [2n + 2]


= n (n + 1)


∴ Sum of first n even natural numbers is n(n + 1).



Question 19.

The first term of an AP is p and its common difference is q. Find its 10th term.


Answer:

Here, given: first term = p

Common difference = q


To find: a10


a10 = a + (10 - 1)d


⇒ a10 = p + 9q


∴ 10th term of the given AP will be p + 9q.



Question 20.

If 4/5, a, 2 are in AP, find the value of a.


Answer:

Since, the terms are in an AP, therefore

a - (4/5) = 2 - a


⇒ 2a = 2 + (4/5)


⇒ 2a = 14/5


⇒ a = 14/10


⇒ a = 7/5


∴ a = 7/5



Question 21.

If (2p + 1), 13, (5p – 3) are in AP, find the value of p.


Answer:

Since, the terms are in an AP, therefore

13 - (2p + 1) = (5p - 3) - (13)


⇒ 12 - 2p = 5p - 16


⇒ 7p = 28


⇒ p = 4


∴ p = 4



Question 22.

If (2p – 1), 7, 3p are in AP, find the value of p.


Answer:

Since, the terms are in an AP, therefore

7 - (2p - 1) = 3p - 7


⇒ 8 - 2p = 3p - 7


⇒ 5p = 15


⇒ p = 3


∴ p = 3



Question 23.

If the sum of first p terms of an AP is (ap2 + bp), find its common


Answer:

Let Sp denotes the sum of first p terms of an AP.

Sum of first p terms = Sp = ap2 + bp


Then pth term is given by: ap = Sp - Sp - 1


∴ ap = (ap2 + bp) - [a(p - 1)2 + b(p - 1)]


= (ap2 + bp) - [a(p2 + 1 - 2p) + bp - b]


= ap2 + bp - ap2 - a + 2ap - bp + b


= b - a + 2ap


Now, common difference = d = ap - ap - 1


= b - a + 2ap - [b - a + 2a(p - 1)]


= b - a + 2ap - b + a - 2ap + 2a


= 2a


∴ common difference = 2a


ALITER: Let Sp denotes the sum of first p terms of an AP.


Sum of first p terms = Sp = ap2 + bp


Put p = 1, we get S1 = a + b


Put p = 2, we get S2 = 4a + 2b


Now S1 = a1


a2 = S2 - S1


∴ a2 = 3a + b


Now, d = a2 - a1


= 3a + b - (a + b)


= 2a


∴ Common difference = 2a



Question 24.

If the sum of first n terms is (3n2 + 5n), find its common difference.


Answer:

Let Sn denotes the sum of first n terms of an AP.

Sum of first n terms = Sn = 3n2 + 5n


Then nth term is given by: an = Sn - Sn - 1


∴ an = (3n2 + 5n) - [3(n - 1)2 + 5(n - 1)]


= (3n2 + 5n) - [3(n2 + 1 - 2n) + 5n - 5]


= 3n2 + 5n - 3n2 - 3 + 6n - 5n + 5


= 2 + 6n


Now, common difference = d = an - an - 1


= 2 + 6n - [2 + 6(n - 1)]


= 2 + 6n - 2 - 6n + 6


= 6


∴ Common difference = 6


ALITER: Let Sn denotes the sum of first n terms of an AP.


Sum of first n terms = Sn = 3n2 + 5n


Put n = 1, we get S1 = 8


Put n = 2, we get S2 = 22


Now S1 = a1


a2 = S2 - S1


∴ a2 = 22 - 8 = 14


Now, d = a2 - a1


= 14 - 8


= 6


∴ Common difference = 6



Question 25.

Find an AP whose 4th term is 9 and the sum of its 6th and 13th terms is 40.


Answer:

Let a be the first term and d be the common difference.

Given: a4 = 9


a6 + a13 = 40


Now, Consider a4 = 9


⇒ a + (4 - 1)d = 9


⇒ a + 3d = 9 ……….(1)


Consider a6 + a13 = 40


⇒ a + (6 - 1)d + a + (13 - 1)d = 40


⇒ 2a + 17d = 40 ………..(2)


Subtracting twice of equation (1) from equation (2), we get,


11d = 22


⇒ d = 2


∴ Common difference = d = 2


Now from equation (1),we get


a = 9 - 3d


= 9 - 6


= 3


∴ AP is a, a + d, a + 2d, a + 3d, …


i.e. AP is 3, 5, 7,9, 11…..




Exercise 11d
Question 1.

Find the sum of each of the following APs:

2, 7, 12, 17, ... to 19 terms.


Answer:

Here, first term = 2

Common difference = 7 - 2 = 5


Sum of first n terms of an AP is


Sn = [2a + (n - 1)d]


∴ S19 = [2(2) + (19 - 1)5]


= (19)(4 + 90)/2


= (19 × 94)/2


= 893


Thus, sum of 19 terms of this AP is 893.



Question 2.

Find the sum of each of the following APs:

9, 7, 5, 3, ... to 14 terms.


Answer:

Here, first term = 9

Common difference = 7 - 9 = - 2


Sum of first n terms of an AP is


Sn = [2a + (n - 1)d]


∴ S14 = [2(9) + (14 - 1)(-2)]


= (7)(18 - 26)


= (7) × (-8)


= - 56


Thus, sum of 14 terms of this AP is - 56.



Question 3.

Find the sum of each of the following APs:

– 37, – 33, – 29, ... to 12 terms.


Answer:

Here, first term = - 37

Common difference = (-33) - (-37) = 4


Sum of first n terms of an AP is


Sn = [2a + (n - 1)d]


∴ S12 = [2(-37) + (12 - 1)(4)]


= (6)(-74 + 44)


= 6 × (-30)


= - 180


Thus, sum of 12 terms of this AP is - 180.



Question 4.

Find the sum of each of the following APs:

(1/15), (1/12), (1/10),…….. to 11 terms.


Answer:

Here, first term = 1/15

Common difference = (1/12) - (1/15) = 1/60


Sum of first n terms of an AP is


Sn = [2a + (n - 1)d]


∴ S11 = [2(1/15) + (11 - 1)(1/60)]


= (11/2) × [(2/15) + (1/6)]


= (11/2) × [(3/10)]


= 33/20


Thus, sum of 11 terms of this AP is 33/20.



Question 5.

Find the sum of each of the following APs:

0.6, 1.7, 2.8, ... to 100 terms.


Answer:

Here, first term = 0.6

Common difference = 1.7 - 0.6 = 1.1


Sum of first n terms of an AP is


Sn = [2a + (n - 1)d]


∴ S100 = [2(0.6) + (100 - 1)(1.1)]


= (50) × [1.2 + (99 × 1.1) ]


= 50 × [1.2 + 108.9]


= 50 × 110.1


= 5505


Thus, sum of 100 terms of this AP is 5505.



Question 6.

Find the sum of each of the following arithmetic series:



Answer:

Here, First term = 7

Common difference = d = (21/2) - 7 = (7/2)


Last term = l = 84


Now, 84 = a + (n - 1)d


∴ 84 = 7 + (n - 1)(7/2)


⇒ 84 - 7 = (n - 1)(7/2)


⇒ 77 = (n - 1)(7/2)


⇒ 154 = 7n - 7 (multiplying both sides by 2)


⇒ 154 + 7 = 7n


⇒ 7n = 161


⇒ n = 23


∴ there are 23 terms in this Arithmetic series.


Now, Sum of these 23 terms is given by


∴ S23 = [2(7) + (23 - 1)(7/2)]


= (23/2) × [14 + (22)(7/2) ]


= (23/2) × [14 + 77]


= (23/2) × [91]


= 2093/2


= 1046.5


Thus, sum of 23 terms of this AP is 1046.5.



Question 7.

Find the sum of each of the following arithmetic series:

34 + 32 + 30 + … + 10.


Answer:

Here, First term = 34

Common difference = d = 34 - 32 = - 2


Last term = l = 10


Now, 10 = a + (n - 1)d


∴ 10 = 34 + (n - 1)(-2)


⇒ 10 - 34 = (n - 1)(-2)


⇒ - 24 = - 2n + 2


⇒ - 24 - 2 = - 2n


⇒ - 26 = - 2n


⇒ n = 13


⇒ n = 13


∴ there are 13 terms in this Arithmetic series.


Now, Sum of these 13 terms is given by


∴ S13 = [2(34) + (13 - 1)(-2)]


= (13/2) × [68 + (12)(-2) ]


= (13/2) × [68 - 24]


= (13/2) × [44]


= 13 × 22


= 286


Thus, sum of 23 terms of this AP is 286.



Question 8.

Find the sum of each of the following arithmetic series:

(-5) + (-8) + (-11) + … + (-230)


Answer:

Here, First term = - 5

Common difference = d = - 8 - (-5) = - 3


Last term = l = - 230


Now, - 230 = a + (n - 1)d


∴ - 230 = - 5 + (n - 1)(-3)


⇒ - 230 + 5 = (n - 1)(-3)


⇒ - 225 = - 3n + 3


⇒ - 225 - 3 = - 3n


⇒ - 228 = - 3n


⇒ n = 76


∴ there are 76 terms in this Arithmetic series.


Now, Sum of these 76 terms is given by


∴ S76 = [2(-5) + (76 - 1)(-3)]


= 38 × [ - 10 + (75)(-3) ]


= 38 × [ - 10 - 225]


= 38 × (-235)


= - 8930


Thus, sum of 23 terms of this AP - 8930.



Question 9.

Find the sum of first n terms of an AP whose nth term is (5 - 6n). Hence, find the sum of its first 20 terms.


Answer:

Since, nth term is given as (5 - 6n)

Put n = 1, we get a1 = - 1 = first term


Put n = 2, we get a2 = - 7 = second term


Now, d = a2 - a1 = - 7 - (-1) = - 6


Sum of first n terms = Sn= [2a + (n - 1)d]; where a is the first term


and d is the common difference.


= [ - 2 + (n - 1)(-6)]


= n[ - 1 - 3n + 3]


= n(2 - 3n)


∴ sum of first 20 terms = S20


= [2(-1) + (20 - 1)(-6)]


= 10 × [ - 2 - 114]


= 10 × [ - 116]


= - 1160



Question 10.

The sum of the first n terms of an AP is (3n2 + 6n). Find the nth term and the 15th term of this AP.


Answer:

Given: The sum of the first n terms of an AP is (3n2 + 6n).

To find: the nth term and the 15th term of this AP.

Solution:

Sum of first n terms = Sn = 3n2 + 6n

Now let an be the nth term of the AP.

To find: an and a15

Since an =Sn - Sn - 1


⇒ an= (3n2 + 6n) - (3(n - 1)2 + 6(n - 1))

⇒ an= (3n2 + 6n) - (3(n2 +1 - 2n) + 6(n - 1))

⇒ an = (3n2 + 6n) - (3n2 + 3 - 6n + 6n - 6)

⇒ an = 3n2 + 6n - 3n2 - 3 + 6n - 6n + 6

⇒ an = 6n + 3


Now, a15 = 6(15) + 3


⇒ a15= 93


Question 11.

The sum of the first n terms of an AP is given by Sn = (3n2 - n) . Find its

(i) nth term,

(ii) first term and

(iii) Common difference.


Answer:

(i) Let an be the nth term of the AP.

To find: an


Then an =Sn - Sn - 1


= (3n2 - n) - (3(n - 1)2 - (n - 1))


= (3n2 - n) - (3n2 + 3 - 6n - n + 1)


= 6n - 4


(ii) Since an = 6n - 4


∴ For first term, n = 1


By putting n = 1 in the nth term, we get,


a1 = 6(1) - 4


= 2


∴ a = 2


(iii) Put n = 2 in the nth term, we get


a2 = 6 × (2) - 4


= 12 - 4


= 8


Now common difference = d = a2 - a1


= 8 - 2


= 6


∴ Common difference = 6



Question 12.

The sum of the first n terms of an AP is . Find the nth term and the 20th term of this AP.


Answer:

Let an be the nth term of the AP.

To find: an and a20


Since, an =Sn - Sn - 1


= ( ) - ()


= 1/2 (5n2 + 3n) - 1/2 [5(n - 1)2 + 3(n - 1)]


= 1/2 (5n2 + 3n - 5n2 - 5 + 10n - 3n + 3)


= 1/2 (10n - 2)


= 5n - 1


Since an = 5n - 1


∴ For 20th term, put n = 20, we get,


a20 = 5(20) - 1


= 100 - 1


= 99



Question 13.

The sum of the first n terms of an AP is . Find its nth term and the 25th term.


Answer:

Let an be the nth term of the AP.

To find: an and a25


Since, an =Sn - Sn - 1


= ( ) - ()


= 1/2 (3n2 + 5n) - 1/2 [3(n - 1)2 + 5(n - 1)]


= 1/2 (3n2 + 5n - 3n2 - 3 + 6n - 5n + 5)


= 1/2 (6n - 2)


= 3n + 1


Since an = 5n - 1


∴ For 25th term, put n = 25, we get,


a25 = 3(25) + 1


= 75 + 1


= 76



Question 14.

How many terms of the AP 21, 18, 15, ... must be added to get the sum 0?


Answer:

Here, first term = a = 21

Common difference = d = 18 - 21 = - 3


Let first n terms of the AP sums to zero.


∴ Sn = 0


To find: n


Now, Sn = (n/2) × [2a + (n - 1)d]


Since, Sn = 0


∴ (n/2) × [2a + (n - 1)d] = 0


⇒ (n/2) × [2(21) + (n - 1)(-3)] = 0


⇒ (n/2) × [42 - 3n + 3)] = 0


⇒ (n/2) × [45 - 3n] = 0


⇒ [45 - 3n] = 0


⇒ 45 = 3n


⇒ n = 15


∴ 15 terms of the given AP sums to zero.



Question 15.

How many terms of the AP 9, 17, 25, ... must be taken so that their sum is 636?


Answer:

Here, first term = a = 9

Common difference = d = 17 - 9 = 8


Let first n terms of the AP sums to 636.


∴ Sn = 636


To find: n


Now, Sn = (n/2) × [2a + (n - 1)d]


Since, Sn = 636


∴ (n/2) × [2a + (n - 1)d] = 636


⇒ (n/2) × [2(9) + (n - 1)(8)] = 636


⇒ (n/2) × [18 + 8n - 8)] = 636


⇒ (n/2) × [10 + 8n] = 636


⇒ n[5 + 4n] = 636


⇒ 4n2 + 5n - 636 = 0


⇒ 4n2 + 5n - 636 = 0


⇒ (n - 12)(4n + 53) = 0


⇒ n = 12 or n = - 53/4


But n can’t be negative and fraction.


∴ n= 12


∴ 12 terms of the given AP sums to 636.



Question 16.

How many terms of the AP 63, 60, 57, 54, ... must be taken so that their sum is 693? Explain the double answer.


Answer:

Here, first term = a =63

Common difference = d = 60 - 63 = - 3


Let first n terms of the AP sums to 693.


∴ Sn = 693


To find: n


Now, Sn = (n/2) × [2a + (n - 1)d]


Since, Sn = 693


∴ (n/2) × [2a + (n - 1)d] = 693


⇒ (n/2) × [2(63) + (n - 1)(-3)] = 693


⇒ (n/2) × [126 - 3n + 3)] = 693


⇒ (n/2) × [129 - 3n] = 693


⇒ n[129 - 3n] = 1386


⇒ 129n - 3n2 = 1386


⇒ 3n2 - 129n + 1386 = 0


⇒ (n - 22)( n - 21)= 0


⇒ n = 22 or n = 21


∴ n= 22 or n = 21


Since, a22 = a + 21d


= 63 + 21(-3)


= 0


∴ Both the first 21 terms and 22 terms give the sum 693 because the 22nd term is 0. So, the sum doesn’t get affected.



Question 17.

How many terms of the AP ,... must be taken so that their sum is 300? Explain the double answer.


Answer:

Here, first term = a =20

Common difference = d = 58/3 - 20 = - 2/3


Let first n terms of the AP sums to 300.


∴ Sn = 300


To find: n


Now, Sn = (n/2) × [2a + (n - 1)d]


Since, Sn = 300


∴ (n/2) × [2a + (n - 1)d] = 300


⇒ (n/2) × [2(20) + (n - 1)(-2/3)] = 300


⇒ (n/2) × [40 - (2/3)n + (2/3)] = 300


⇒ (n/2) × [(120 - 2n + 2)/3] = 300


⇒ n[122 - 2n] = 1800


⇒ 122n - 2n2 = 1800


⇒ 2n2 - 122n + 1800 = 0


⇒ n2 - 61n + 900 = 0


⇒ (n - 36)( n - 25)= 0


⇒ n = 36 or n = 25


∴ n= 36 or n = 25


Now, S36 = (36/2)[2a + 35d]


= 18(40 + 35(-2/3))


= 18(120 - 70)/3


= 6(50)


= 300


Also, S25 = (25/2)[2a + 24d]


= (25/2)(40 + 24(-2/3))


= (25/2)(40 - 16)


= (24 × 25)/2


= 12 × 25


= 300


Now, sum of 11 terms from 26th term to 36th term = S36 - S25 = 0


∴ Both the first 25 terms and 36 terms give the sum 300 because the sum of last 11 terms is 0. So, the sum doesn’t get affected.



Question 18.

Find the sum of all odd numbers between 0 and 50.


Answer:

Odd numbers from 0 to 50 are 1, 3, 5, …, 49

Sum of these numbers is 1 + 3 + 5 + … + 49.


This forms an Arithmetic Series with first term = a = 1


and Common Difference = d = 3 - 1 = 2


There are 25 terms in this Arithmetic Series.


Now, sum of n terms is given as:


Sn = (n/2)[2a + (n - 1)d]


S25 = (25/2)[2(1) + (25 - 1)2]


= (25/2)[2 + 48]


= (25 × 50)/2


= 25 × 25


= 625


∴ Sum of odd numbers from 0 to 50 is 625.



Question 19.

Find the sum of all natural numbers between 200 and 400 which are divisible by 7.


Answer:

Natural numbers between 200 and 400 which are divisible by 7 are 203, 210, 217, …, 399.

Sum of these numbers forms an arithmetic series 203 + 210 + 217 + … + 399.


Here, first term = a = 203


Common difference = d = 7


∴ an = a + (n - 1)d


⇒ 399 = 203 + (n - 1)7


⇒ 399 = 7n + 196


⇒ 7n = 203


⇒ n = 29


∴ there are 29 terms in the AP.


Sum of n terms of this arithmetic series is given by:


Sn = [2a + (n - 1)d]


Therefore sum of 28 terms of this arithmetic series is given by:


∴ S29 = [2(203) + (29 - 1)(7)]


= (29/2) [406 + 196]


=(29/2) × 502


= 7279



Question 20.

Find the sum of first forty positive integers divisible by 6.


Answer:

First 40 positive integers divisible by 6 are 6, 12, 18, …, 240.

Sum of these numbers forms an arithmetic series 6 + 12 + 18 + … + 240.


Here, first term = a = 6


Common difference = d = 6


Sum of n terms of this arithmetic series is given by:


Sn = [2a + (n - 1)d]


Therefore sum of 40 terms of this arithmetic series is given by:


∴ S40 = [2(6) + (40 - 1)(6)]


= 20 [12 + 234]


=20 × 246


= 4920



Question 21.

Find the sum of the first 15 multiples of 8.


Answer:

First 15 multiples of 8 are 8, 16, 24, …, 120.

Sum of these numbers forms an arithmetic series 8 + 16 + 24 + … + 120.


Here, first term = a = 8


Common difference = d = 8


Sum of n terms of this arithmetic series is given by:


Sn = [2a + (n - 1)d]


Therefore sum of 15 terms of this arithmetic series is given by:


∴ S15 = [2(8) + (15 - 1)(8)]


= (15/2) [16 + 112]


=(15/2) × 128


= 15 × 64


= 960



Question 22.

Find the sum of all multiples of 9 lying between 300 and 700.


Answer:

Multiples of 9 lying between 300 and 700 are 306, 315, 324, …, 693.

Sum of these numbers forms an arithmetic series 306 + 315 + 324 + … + 693.


Here, first term = a = 306


Common difference = d = 9


We first find the number of terms in the series.


Here, last term = l = 693


∴ 693 = a + (n - 1)d


⇒ 693 = 306 + (n - 1)9


⇒ 693 - 306 = 9n - 9


⇒ 387 = 9n - 9


⇒ 387 + 9 = 9n


⇒ 9n = 396


⇒ n = 44


Now, Sum of n terms of this arithmetic series is given by:


Sn = [2a + (n - 1)d]


Therefore sum of 44 terms of this arithmetic series is given by:


∴ S44 = [2(306) + (44 - 1)(9)]


= 22 × [612 + 387]


= 22 × 999


= 21978



Question 23.

Find the sum of all three - digit natural numbers which are divisible by 13.


Answer:

Three - digit natural numbers which are divisible by 13 are 104, 117, 130, …, 988.

Sum of these numbers forms an arithmetic series 104 + 117 + 130 + … + 988.


Here, first term = a = 104


Common difference = d = 13


We first find the number of terms in the series.


Here, last term = l = 988


∴ 988 = a + (n - 1)d


⇒ 988 = 104 + (n - 1)13


⇒ 988 - 104 = 13n - 13


⇒ 884 = 13n - 13


⇒ 884 + 13 = 13n


⇒ 13n = 897


⇒ n = 69


Now, Sum of n terms of this arithmetic series is given by:


Sn = [2a + (n - 1)d]


Therefore sum of 69 terms of this arithmetic series is given by:


∴ S69 = [2(104) + (69 - 1)(13)]


= (69/2) × [208 + 884]


= (69/2) × 1092


= 69 × 546


= 3767



Question 24.

Find the sum of first 100 even natural numbers which are divisible by 5.


Answer:

First 100 even natural numbers which are divisible by 5 are 10, 20, 30, …, 1000

Here, first term = a = 10


Common difference = d = 10


Number of terms = 100


Now, Sum of n terms of this arithmetic series is given by:


Sn = [2a + (n - 1)d]


Therefore sum of 100 terms of this arithmetic series is given by:


∴ S100 = [2(10) + (100 - 1)(10)]


= 50 × [20 + 990]


= 50 × 1010


= 50500



Question 25.

Find the sum of the following.

up to n terms.


Answer:

The given sum can be written as (1 + 1 + 1 + …) - (1/n, 2/n, 3/n, …)

Sum of first series up to n terms = 1 + 1 + 1 + … up to n terms


= n


Now, consider the second series:


Here, first term = a = 1/n


Common difference = d = (2/n) - (1/n) = (1/n)


Now, Sum of n terms of an arithmetic series is given by:


Sn = [2a + (n - 1)d]


Therefore sum of n terms of second arithmetic series is given by:


∴ Sn = [2(1/n) + (n - 1)(1/n)]


= [(2/n) + 1 - (1/n)]


= [(1/n) + 1]


= = = (n + 1)/2


Now, sum of n terms of the complete series = Sum of n terms of first series - Sum of n terms of second series


= n - (n + 1)/2


= (2n - n - 1)/2


= 1/2 (n - 1)



Question 26.

In an AP, it is given that, S5 + S7 =167 and S10= 235, then find the AP, where Sn denotes the sum of its first n terms.


Answer:

Let the first term be a.

Let Common difference be d.


Given: S5 + S7 =167


S10= 235


Now, Sum of n terms of an arithmetic series is given by:


Sn = [2a + (n - 1)d]


So, consider


S5 + S7 =167


⇒ (5/2) [2a + (5 - 1)d] + (7/2) [2a + (7 - 1)d] = 167


⇒ (5/2) [2a + 4d] + (7/2) [2a + 6d] = 167


⇒ 5 × [a + 2d] + 7 × [a + 3d] = 167


⇒ 5a + 10d + 7a + 21d = 167


⇒ 12a + 31d = 167 …………….. (1)


Now, consider S10= 235


⇒ (10/2) [2a + (10 - 1)d] = 235


⇒ 5 × [2a + 9d] = 235


⇒ 10a + 45d = 235


⇒ 2a + 9d = 47 ………. (2)


Subtracting equation (1) from 6 times of equation (2), we get,


⇒ 23d = 115


⇒ d = 5


So, from equation (2), we get,


a = 1/2 (47 - 9d)


⇒ a = 1/2 (47 - 45)


⇒ a = 1/2 (2)


⇒ a = 1


Therefore the AP is a, a + d, a + 2d, a + 3d,…


i.e. 1, 6, 11, 16, ….



Question 27.

In an AP, the first term is 2, the last term is 29 and the sum of all the terms is 155. Find the common difference.


Answer:

Here, first term = a = 2

Let the Common difference = d


Last term = l = 29


Sum of all terms = Sn = 155


Let there be n terms in the AP.


Now, Sum of n terms of this arithmetic series is given by:


Sn = [2a + (n - 1)d]


= [a + a + (n - 1)d]


= [a + l]


Therefore sum of n terms of this arithmetic series is given by:


∴ Sn = [2 + 29] = 155


⇒ 31n = 310


⇒ n = 10


∴ there are 10 terms in the AP.


Thus 29 be the 10th term of the AP.


∴ 29 = a + (10 - 1)d


⇒ 29 = 2 + 9d


⇒ 27 = 9d


⇒ d = 3


∴ common difference = d =3



Question 28.

In an AP, the first term is – 4, the last term is 29 and the sum of all its terms is 150. Find its common difference.


Answer:

Here, first term = a = - 4

Let the Common difference = d


Last term = l = 29


Sum of all terms = Sn = 150


Let there be n terms in the AP.


Now, Sum of n terms of this arithmetic series is given by:


Sn = [2a + (n - 1)d]


= [a + a + (n - 1)d]


= [a + l]


Therefore sum of n terms of this arithmetic series is given by:


∴ Sn = [ - 4 + 29] = 150


⇒ 25n = 300


⇒ n = 12


∴ there are 12 terms in the AP.


Thus 29 is the 12th term of the AP.


∴ 29 = a + (12 - 1)d


⇒ 29 = - 4 + 11d


⇒ 29 + 4 = 11d


⇒ 11d = 33


⇒ d = 3


∴ Common difference = d =3



Question 29.

The first and the last terms of an AP are 17 and 350 respectively. If the common difference is 9, how many terms are there and what is their sum?


Answer:

Here, first term = a = 17

Common difference = 9


Last term = l = 350


To find: number of terms and their sum.


Let there be n terms in the AP.


Since, l= 350


∴ 350 = 17 + (n - 1)9


⇒ 350 - 17 = 9n - 9


⇒ 333 = 9n - 9


⇒ 333 + 9 = 9n


⇒ 9n = 342


⇒ n = 38


Therefore number of terms = 38


Now, Sum of n terms of this arithmetic series is given by:


Sn = [2a + (n - 1)d]


= [a + a + (n - 1)d]


= [a + l]


Therefore sum of 38 terms of this arithmetic series is given by:


∴ S38 = [17 + 350]


= 19 × 367


= 6973


∴ n= 38 and Sn = 6973



Question 30.

The first and the last terms of an AP are 5 and 45 respectively. If the sum of all its terms is 400, find the common difference and the number of terms.


Answer:

Here, first term = a = 5

Let the Common difference = d


Last term = l = 45


Sum of all terms = Sn = 400


Let there be n terms in the AP.


Now, Sum of n terms of this arithmetic series is given by:


Sn = [2a + (n - 1)d]


= [a + a + (n - 1)d]


= [a + l]


Therefore sum of n terms of this arithmetic series is given by:


∴ Sn = [5 + 45] = 400


⇒ 50n = 800


⇒ n = 16


∴ there are 16 terms in the AP.


Thus 45 is the 16th term of the AP.


∴ 45 = a + (16 - 1)d


⇒ 45 = 5 + 15d


⇒ 40 = 15d


⇒ 15d = 40


⇒ d = 8/3


∴ Common difference = d = 8/3



Question 31.

In an AP, the first term is 22, nth term is – 11 and sum of first n terms is 66. Find n and hence find the common difference.


Answer:

Here, first term = a = 22

Let the Common difference = d


nth term = an = - 11


Sum of first n terms = Sn = 66


Let there be n terms in the AP.


Now, Sum of n terms of this arithmetic series is given by:


Sn = [2a + (n - 1)d]


= [a + a + (n - 1)d]


= [a + an]


Therefore sum of n terms of this arithmetic series is given by:


∴ Sn = [22 + (-11)] = 66


⇒ 11n = 132


⇒ n = 12


∴ there are 12 terms in the AP.


Thus nth is the 12th term of the AP.


∴ - 11 = a + (12 - 1)d


⇒ - 11 = 22 + 11d


⇒ - 11 - 22 = 11d


⇒ 11d = - 33


⇒ d = - 3


∴ Common difference = d = - 3


∴ n = 12, d = - 3



Question 32.

The 12th term of an AP is – 13 and the sum of its first four terms is 24. Find the sum of its first 10 terms.


Answer:

Let a be the first term and d be the common difference.

Given: a12 = - 13


S4 = 24


To find: Sum of first 10 terms.


Consider a12 = - 13


⇒ a + 11d = - 13 ………………(1)


Also, S4 = 24


⇒ (4/2) × [2a + (4 - 1)d] = 24


⇒ 2 × [2a + 3d] = 24


⇒ 2a + 3d = 12 …………….(2)


Subtracting equation (2) from twice of equation (1), we get,


19d = - 38


⇒ d = - 2


Now, from equation (1), we get


a = - 13 - 11d


⇒ a = - 13 - 11(-2)


⇒ a = - 13 + 22


⇒ a = 9


Now, Sum of first n terms of this arithmetic series is given by:


Sn = [2a + (n - 1)d]


Therefore sum of first 10 terms of this arithmetic series is given by:


∴ S10 = [2(9) + (10 - 1)(-2)]


= 5 × [18 - 18]


= 0


∴ S10 = 0



Question 33.

The sum of the first 7 terms of an AP is 182. If its 4th and 17th terms are in the ratio 1 : 5, find the AP.


Answer:

Let a be the first term and d be the common difference.

Given: S7 = 182


4th and 17th terms are in the ratio 1 : 5.


i.e. [a + 3d] : [(a + 16d] = 1 : 5


=


⇒ 5(a + 3 d) = (a + 16d)


⇒ 5a + 15d = a + 16d


⇒ 4a = d


Now, consider S7 = 182


⇒ (7/2)[2a + (7 - 1)d] = 182


⇒ (7/2)[2a + 6(4a)] = 182


⇒ 7 × [26a] = 182 × 2


⇒ 182a = 364


⇒ a = 2


∴ d = 4a


⇒ d = 8


Thus the AP will be a, a + d, a + 2d,…


i.e. AP is 2, 10, 18, 26,….



Question 34.

The sum of the first 9 terms of an AP is 81 and that of its first 20 terms is 400. Find the first term and the common difference of the AP.


Answer:

Let a be the first term and d be the common difference.

Given: S9 = 81, S20 = 400


Now, consider S9 = 81


⇒ (9/2)[2a + (9 - 1)d] = 81


⇒ (9/2)[2a + 8d] = 81


⇒ [2a + 8d] = 18 ………(1)


Now, consider S20 = 400


⇒ (20/2)[2a + (20 - 1)d] = 400


⇒ 10 × [2a + 19d] = 400


⇒ [2a + 19d] = 40 ………..(2)


Now, on subtracting equation (2) from equation (1), we get,


11d = 22


⇒ d = 2


∴ from equation (1), we get


a = 1/2 (18 - 8d)


⇒ a = 9 - 4d


⇒ a = 9 - 8


⇒ a = 1


∴ a = 1, d = 2



Question 35.

The sum of the first 7 terms of an AP is 49 and the sum of its first 17 terms is 289. Find the sum of its first n terms.


Answer:

Let a be the first term and d be the common difference.

Given: S7 = 49, S17 = 289


To find: sum of first n terms.


Now, consider S7 = 49


⇒ (7/2)[2a + (7 - 1)d] = 49


⇒ (7/2)[2a + 6d] = 49


⇒ [a + 3d] = 7 …………(1)


Now, consider S17 = 289


⇒ (17/2)[2a + (17 - 1)d] = 289


⇒ (17/2) × [2a + 16d] = 289


⇒ [a + 8d] = 17 …………..(2)


Now, on subtracting equation (2) from equation (1), we get,


5d = 10


⇒ d = 2


∴ from equation (1), we get


a = (7 - 3d)


⇒ a = 7 - 6


⇒ a = 1


∴ a = 1, d = 2


Now, Sum of first n terms = Sn = (n/2)[2a + (n - 1)d]


= (n/2)[2 + (n - 1)2]


= (n/2)[2n]


= n2


∴ Sn = n2



Question 36.

Two APs have the same common difference. If the first terms of these APs be 3 and 8 respectively, find the difference between the sums of their first 50 terms.


Answer:

Let a1 and a2 be the first terms of the two APs

Let d1 and d2 be the common difference of the respective APs.


Given: d1 = d2 and a1 = 3, a2 = 8


To find: Difference between the sums of their first 50 terms.


i.e. to find: (S2)50 - (S1)50


where (S1)50 denotes the sum of first 50 terms of first AP and (S2)50


denotes the sum of first 50 terms of second AP.


Now, consider (S1)50 = (50/2)[2a1 + (50 - 1)d1]


= 25 × [2(3) + 49 × d1]


= 25[6 + 49d1]


= 150 + 1225d1


Now, consider (S2)50 = (50/2)[2a2 + (50 - 1)d2]


= 25 × [2(8) + 49 × d2]


= 25[16 + 49d1]


= 400 + 1225d2


Now, (S2)50 - (S1)50 = 400 + 1225d2 - (150 + 1225d2)


= 400 - 150 (∵ d1 = d2 )


= 250


∴ (S2)50 - (S1)50 = 250



Question 37.

The sum of first 10 terms of an AP is – 150 and the sum of its next 10 terms is – 550. Find the AP.


Answer:

Let a be the first term and d be the common difference.

Given: Sum of first 10 terms = S10 = - 150


Sum of next 10 terms = - 550


i.e. S20 - S10 = - 550


Consider S10 = - 150


⇒ (10/2)[2a + (10 - 1)d] = - 150


⇒ 5 × [2a + 9d] = - 150


⇒ [2a + 9d] = - 30 ………………….(1)


Now, consider S20 - S10 = - 550


⇒ (20/2)[2a + (20 - 1)d] - (10/2)[2a + (10 - 1)d] = - 550


⇒ 10 × [2a + 19d] - 5[2a + 9d] = - 550


⇒ 10a + 145d = - 550 …………………..(2)


On subtracting equation (2) from 5 times of equation (2), we get,


- 100d = 400


⇒ d = - 4


∴ a = 1/2 (-30 - 9d)


⇒ a = 1/2 (-30 + 36)


⇒ a = 3


Therefore the AP is 3, - 1, - 5, - 9,….



Question 38.

The 13th term of an AP is 4 times its 3rd term. If its 5th term is 16, find the sum of its first 10 terms.


Answer:

Let a be the first term and d be the common difference.

Given: a5 = 16


a13 = 4 a3


Now, Consider a5 = 16


⇒ a + (5 - 1)d = 16


⇒ a + 4d = 16 ………………….(1)


Consider a13 = 4 a3


⇒ a + 12d = 4(a + 2d)


⇒ a + 12d = 4a + 8d


⇒ 3a - 4d = 0 ………………….(2)


Now, adding equation (1) and (2), we get,


4a = 16


⇒ a = 4


∴ from equation (2), we get,


4d = 3a


⇒ 4d = 12


⇒ d = 3


Now, Sum of first n terms of an AP is


Sn = [2a + (n - 1)d]


∴ Sum of first 10 terms is given by:


S10 = [2(4) + (10 - 1)(3)]


= 5 × [8 + 27]


= 5 × 35


= 175


∴ S10=175



Question 39.

The 16th term of an AP is 5 times its 3rd term. If its 10th term is 41, find the sum of its first 15 terms.


Answer:

Let a be the first term and d be the common difference.

Given: a10 = 41


a16 = 5 a3


Now, Consider a10 = 41


⇒ a + (10 - 1)d = 41


⇒ a + 9d = 41 ………………….(1)


Consider a16 = 5 a3


⇒ a + 15d = 5(a + 2d)


⇒ a + 15d = 5a + 10d


⇒ 4a - 5d = 0 ………………….(2)


Now, subtracting equation (2) from 4 times of equation (1), we get,


41d = 164


⇒ d = 4


∴ from equation (2), we get,


4a= 5d


⇒ 4a = 20


⇒ a = 5


Now, Sum of first n terms of an AP is


Sn = [2a + (n - 1)d]


∴ Sum of first 15 terms is given by:


S15 = [2(5) + (15 - 1)(4)]


= (15/2) × [10 + 56]


= 15 × 33


= 495


∴ S15 = 495



Question 40.

An AP 5, 12, 19, ... has 50 terms. Find its last term. Hence, find the sum of its last 15 terms.


Answer:

Here, First term = a = 5

Common difference = d = 12 - 5 = 7


No. of terms = 50


∴ last term will be 50th term.


Using the formula for finding nth term of an A.P.,


l = a50 = a + (50 - 1) × d


l = 5 + (50 - 1) × 7


l = 5 + 343 = 348


Now, sum of last 15 terms = sum of first 50 terms - sum of first 35 terms


i.e. sum of last 15 terms = S50 - S35


Now, Sum of first n terms of an AP is


Sn = [2a + (n - 1)d]


∴ Sum of first 50 terms is given by:


S50 = [2(5) + (50 - 1)(7)]


= 25 × [10 + 343]


= 25 × 353


= 8825


Now, Sum of first 35 terms is given by:


S35 = [2(5) + (35 - 1)(7)]


= (35/2) × [10 + 238]


= (35/2) × 248


= 35 × 124


= 4340


Now, S50 - S35 = 8825 - 4340


= 4485


∴ last term = 348, sum of last 15 terms = 4485



Question 41.

An AP 8, 10, 12, ... has 60 terms. Find its last term. Hence, find the sum of its last 10 terms.


Answer:

Here, First term = a = 8

Common difference = d = 10 - 8 = 2


No. of terms = 60


∴ last term will be 60th term.


Using the formula for finding nth term of an A.P.,


l = a60 = a + (60 - 1) × d


l = 8 + (60 - 1) × 2


l = 8 + 118 = 126


Now, sum of last 10 terms = sum of first 60 terms - sum of first 50 terms


i.e. sum of last 10 terms = S60 - S50


Now, Sum of first n terms of an AP is


Sn = [2a + (n - 1)d]


∴ Sum of first 50 terms is given by:


S50 = [2(8) + (50 - 1)(2)]


= 25 × [16 + 98]


= 25 × 114


= 2850


Now, Sum of first 60 terms is given by:


S60 = [2(8) + (60 - 1)(2)]


= 30 × [16 + 118]


= 30 × 248


= 4020


Now, S60 - S50 = 4020 - 2850


= 1170


∴ last term = 126, sum of last 10 terms = 1170



Question 42.

The sum of the 4th and the 8th terms of an AP is 24 and the sum of its 6th and 10th terms is 44. Find the sum of its first 10 terms.


Answer:

Let a be the first term and d be the common difference.

Given: a4 + a8 = 24


and a6 + a10 = 44


To find: S10


Now, Consider a4 + a8 = 24


⇒ a + 3d + a + 7d= 24


⇒ 2a + 10d = 24 ………….(1)


Consider a6 + a10 = 44


⇒ a + 5d + a + 9d = 44


⇒ 2a + 14d = 44 ………..(2)


Subtracting equation (1) from equation (2), we get,


4d = 20


⇒ d = 5


∴ Common difference = d = 5


Thus from equation (1), we get,


a = - 13


Now, Sum of first n terms of an AP is


Sn = [2a + (n - 1)d]


∴ Sum of first 10 terms is given by:


S10 = [2(-13) + (10 - 1)(5)]


= 5 × [ - 26 + 45]


= 5 × 19


= 95


∴ S10 = 95



Question 43.

The sum of first m terms of an AP is (4m2 - m). If its nth term is 107, find the value of n. Also, find the 21st term of this AP.


Answer:

Let a be the first term and d be the common difference.

Given: Sum of first m terms of an AP is given by:


Sm = [2a + (m - 1)d] = 4m2 - m


Now, nth term is given by: an = Sn - Sn - 1


∴ an = (4n2 - n) - [4(n - 1)2 - (n - 1)]


= (4n2 - n) - [4(n2 + 1 - 2n) - n + 1]


= 4n2 - n - 4n2 - 4 + 8n + n - 1


= 8n - 5 …………………(1)


Now, given that an = 107


⇒ 8n - 5 = 107


⇒ 8n = 112


⇒ n = 14


For 21st term of AP, put n = 21 in the value of the nth term in equation (1), we get


a21 = 8 × (21) - 5


⇒ a21 = 168 - 5


= 163


∴ a21 = 163



Question 44.

The sum of first q terms of an AP is (63q - 3q2). If its pth term is - 60, find the value of p. Also, find the 11th term of its AP.


Answer:

Let a be the first term and d be the common difference.

Given: Sum of first q terms of an AP is given by:


Sq = [2a + (q - 1)d] = 63q - 3q2


Now, pth term is given by: ap = Sp - Sp - 1


∴ ap = (63p - 3p2) - [63(p - 1) - 3(p - 1)2]


= (63p - 3p2) - [63p - 63 - 3p2 - 3 + 6p]


= 63p - 3p2 - 63p + 63 + 3p2 + 3 - 6p


= 66 - 6p …………………(1)


Now, given that ap = - 60


⇒ 66 - 6p = - 60


⇒ 6p = 126


⇒ p = 21


For 11th term of AP, put p = 11 in the value of the pth term in equation (1), we get


a11 = 66 - 6 × (11 )


⇒ a11 = 66 - 66


= 0


∴ a11 = 0



Question 45.

Find the number of terms of the AP - 12, - 9, - 6, ..., 21. If 1 is added to each term of this AP then find the sum of all terms of the AP thus obtained.


Answer:

Here, first term = a = - 12

Common difference = d = - 9 - (-12) = 3


Last term is 21.


Now, number of terms in this AP are given as:


21 = a + (n - 1)d


⇒ 21 = - 12 + (n - 1)3


⇒ 21 + 12 = 3n - 3


⇒ 33 + 3 = 3n


⇒ 36 = 3n


⇒ n = 12


If 1 is added to each term, then the new AP will be - 11, - 8, - 5,…, 22.


Here, first term = a = - 11


Common difference = d = - 8 - (-11) = 3


Last term = l = 22.


Number of terms will be the same,


i.e, number of terms = n = 12


∴ Sum of 12 terms of the AP is given by:


S12 = (12/2) × [a + l]


= 6 × [ - 11 + 22]


= 6 × 11


= 66


∴ Sum of 12 terms of the new AP will be 66.



Question 46.

Sum of the first 14 terms of an AP is 1505 and its first term is 10. Find its 25th term.


Answer:

Here, first term = a = 10

Let the Common difference = d


Sum of first 14 terms = S14 = 1505


Now, Sum of n terms of this arithmetic series is given by:


Sn = [2a + (n - 1)d]


∴ S14 = [2(10) + (14 - 1)d] = 1505


⇒ 7 × [20 + 13d] = 1505


⇒ [20 + 13d] = 215


⇒ 13d = 195


⇒ d = 15


Now, nth term is given by:


∴ an = a + (n - 1)d


⇒ a25 = 10 + (25 - 1)15


= 10 + (24 × 15)


= 10 + 360


= 370



Question 47.

Find the sum of first 51 terms of an AP whose second and third terms are 14 and 18 respectively.


Answer:

Here, second term = a2 = 14

Third term = a3 = 18


∴ Common difference = a3 - a2 = 18 - 14 = 4


Thus first term = a = a2 - d = 14 - 4 = 10


Now, Sum of first n terms of an AP is given by


Sn = [2a + (n - 1)d]


∴ Sum of first 51 terms is given by:


S51 = [2(10) + (51 - 1)(4)]


= (51/2) × [20 + 200]


= (51/2) × 220


= (51) × 110


= 5610


∴ S51 = 5610



Question 48.

In a school, students decided to plant trees in and around the school to reduce air pollution. It was decided that the number of trees that each section of each class will plant will be double of the class in which they are studying. If there are 1 to 12 classes in the school and each class has two sections, find how many trees were planted by students. Which value is shown in the question?


Answer:

Number of trees planted by one section of class 1st = 2

Now, there are 2 sections, ∴ Number of trees planted by class 1st = 4


Number of trees planted by one section of class 2nd = 4


Now, there are 2 sections, ∴ Number of trees planted by class 2nd = 8


This will follow up to class 12th and we will obtain an AP as


4, 8, 12, … upto 12 terms.


Now, Total number of trees planted by the students = 4 + 8 + 12 + … upto 12 terms.


∴ In this Arithmetic series, first term = a = 4


Common difference = d = 4


Now, S12 = (12/2)[2a + (12 - 1)d]


= 6[2(4) + 11(4)]


= 6 × [8 + 44]


= 6 × 52


= 312


∴ Total number of trees planted by the students = 312


Values shown in the question are care and awareness about conservation of nature and environment.



Question 49.

In a potato race, a bucket is placed at the starting point, which is 5 m from the first potato, and the other potatoes are placed 3 m apart in a straight line. There are 10 potatoes in the line. A competitor starts from the bucket, picks up the nearest potato, runs back with it, drops it in the bucket, runs back to pick up the next potato, runs to the bucket to drop it in, and he continues in the same way until all the potatoes are in the bucket. What is the total distance the competitor has to run?



Answer:

To pick the first potato, the competitor has to run 5 m to reach the potato and 5 m to run back to the bucket.

∴ Total distance covered by the competitor to pick first potato = 2 × (5) = 10 m


To pick the second potato, the competitor has to run (5 + 3) m to reach the potato and (5 + 3) m to run back to the bucket.


∴ Total distance covered by the competitor to pick second potato = 2 × (5 + 3) = 16 m


To pick the third potato, the competitor has to run (5 + 3 + 3) m to reach the potato and (5 + 3 + 3) m to run back to the bucket.


∴ Total distance covered by the competitor to pick third potato = 2 × (5 + 3 + 3) = 22 m


This will continue and we will get a sequence of distance as 10, 16, 22,… upto 10 terms (as there are 10 potatoes to pick).


Total distance covered by the competitor to pick all the 10 potatoes = 10 + 16 + 22 + … upto 10 terms.


This forms an Arithmetic series with first term = a = 10


and Common difference = d = 6


Number of terms = n = 10


Now, S10 = (10/2)[2a + (10 - 1)d]


= 5 × [2(10) + 9(6)]


= 5 × [20 + 54]


= 5 × 74


= 370


∴ Total distance covered by the competitor = 370 m



Question 50.

There are 25 trees at equal distances of 5 m in a line with a water tank, the distance of the water tank from the nearest tree being 10 m. A gardener waters all the trees separately, starting from the water tank and returning back to the water tank after watering each tree to get water for the next. Find the total distance covered by the gardener in order to water all the trees.



Answer:

To water the first tree, the gardener has to cover 10 m to reach the tree and 10 m to go back to the tank.

∴ Total distance covered by the gardener to water first tree = 2 × (10) = 20 m


To water the second tree, the gardener has to cover (10 + 5) m to reach the tree and (10 + 5) m to go back to the tank.


∴ Total distance covered by the gardener to water second tree = 2 × (10 + 5) = 30 m


To water the third tree, the gardener has to cover (10 + 5 + 5) m to reach the tree and (10 + 5 + 5) m to go back to the tank.


∴ Total distance covered by the gardener to water third tree = 2 × (10 + 5 + 5) = 40 m


This will continue and we will get a sequence of distance as 20, 30, 40,… upto 25 terms (as there are 25 trees to be watered).


Total distance covered by the gardener to water all 25 trees = 20 + 30 + 40 + … upto 25 terms.


This forms an Arithmetic series with first term = a = 20


and Common difference = d = 10


Number of terms = n = 25


Now, S25 = (25/2)[2a + (25 - 1)d]


= (25/2) × [2(20) + 24(10)]


= (25/2) × [40 + 240]


= (25/2) × 280


= 25 × 140


= 3500


∴ Total distance covered by the gardener = 3500 m



Question 51.

A sum of Rs. 700 is to be used to give seven cash prizes to students of a school for their overall academic performance. If each prize is Rs. 20 less than its preceding prize, find the value of each prize.


Answer:

Let the first prize be Rs. x. Thus each succeeding prize is Rs. 20 less than the preceding prize.

∴ Second prize, third prize, …, seventh prize be Rs. (x - 20), (x - 40) , …, (x - 120).


This forms an AP as x, x - 20, …, x - 120.


Here, first term = x


Common difference = x - 20 - x = - 20


Total number of terms = 7


Since, Total sum of prize amount = 700.


∴ Sum of all the terms = 700


Now, sum of first n terms of an AP is given by:


Sn = [2a + (n - 1)d]


∴ Sum of 7 terms of an AP is given by:


S7 = [2a + (7 - 1)d] = 700


[2x + (7 - 1)(-20)] = 700


⇒ 7[2x - 120] = 1400


⇒ 2x - 120 = 200


x - 60 = 100


x = 160


Thus, the prizes are as Rs. 160, Rs.140, Rs.120, Rs. 100, Rs. 80, Rs. 60, Rs. 40.



Question 52.

A man saved Rs. 33000 in 10 months. In each month after the first, he saved Rs. 100 more than he did in the preceding month. How much did he save in the first month?


Answer:

Let the amount of money the man saved in first month = Rs. x


Now, the amount of money he saved in second month = Rs.(x + 100)


The amount of money he saved in third month = Rs.(x + + 100 + 100)


This will continue for 10 months.


∴ We get a an AP as x , x + 100, x + 200,… up to 10 terms.


Here, first term = x


Common difference = d = 100


Number of terms = n = 10


Total amount of money saved by the man = x + (x + 100) + (x + 200) + … up to 10 terms. = Rs. 33000 (given)


∴ Sum of 10 terms of the Arithmetic Series = 33000


⇒ S10 =33000


⇒ (10/2) × [2a + (10 - 1)d] =33000


⇒ (10/2) × [2(x) + 9(100)] =33000


⇒ 5 × [2x + 900] =33000


⇒ 2x + 900 =6600


⇒ 2x = 6600 - 900


⇒ 2x = 5700


x = 2850


∴ Amount of money saved by the man in first month = Rs. 2850



Question 53.

A man arranges to pay off a debt of Rs. 36000 by 40 monthly instalments which form an arithmetic series. When 30 of the instalments are paid, he dies leaving one - third of the debt unpaid. Find the value of the first instalment.


Answer:

Let the first installment = Rs. x

Since the instalments form an arithmetic series, therefore let the common difference = d


Now, amount paid in 30 installments = two - third of the amount = (2/3) × (36000) = Rs. 24000


∴ Total amount paid by the man in 30 installments = 24000


Let Sn be that amount paid in 30 installments.


∴ S30 = 24000


⇒ (30/2) × [2x + (30 - 1)d] = 24000


⇒ 15 × [2x + 29d] = 24000


⇒ 2x + 29d = 1600 …………..(1)


Now, Total sum of the amount = 36000


∴ S40 = 36000


⇒ (40/2) × [2x + (40 - 1)d] = 36000


⇒ 20 × [2x + 39d] = 36000


⇒ 2x + 39d = 1800 ………..(2)


Subtracting equation (1) from equation (2), we get:


10d = 200


⇒ d = 20


∴ from equation (1), we get


x = 1/2(1600 - 29d)


= 1/2 (1600 - 580)


= 1/2 (1020)


= 510


Therefore the amount of first installment = Rs. 510



Question 54.

A contract on construction job specifies a penalty for delay of completion beyond a certain date as follows: Rs. 200 for the first day, Rs. 250 for the second day, Rs. 300 for the third day, etc., the penalty for each succeeding day being Rs. 50 more than for the preceding day. How much money the contractor has to pay as penalty, if he has delayed the work by 30 days?


Answer:

Penalty for delay for first day = Rs. 200

Penalty for delay for second day = Rs. 250


Penalty for delay for third day = Rs. 300


Penalty for each succeeding day is Rs. 50 more than for the preceding day.


∴ The amount of penalties are in AP with common difference


= d = Rs.50


Also, number of days in delay of the work = 30 days


Thus the penalties are 200, 250, 300, … up to 30 terms


Thus the amount of money paid by the contractor is 200 + 250 + 300 + … up to 30 terms


Here, first term = a = 200


Common difference = d = 50


Number of terms = n = 30


∴ The sum = S30 = (30/2) × [2(200) + (30 - 1)(50)]


= 15 × [400 + 1450]


= 15 × 1850


= 27750


Thus the total amount of money paid by the contractor = Rs. 27750




Multiple Choice Questions (mcq)
Question 1.

The common difference of the AP is
A. p

B. - p

C. - 1

D. 1


Answer:

Common difference = T2 - T1 =


= = - 1


Question 2.

The common difference of the AP is
A.

B.

C. b

D. – b


Answer:

Common difference = T2 - T1 =



Question 3.

The next term of the AP is
A.

B.

C.

D.


Answer:

Here, first term = √7


Common difference = √28 - √7 = 2√7 - √7 = √7


Next term = T4 = T3 + d


= √63 + √7


= 3√7 + √7


= 4√7


= √112


Question 4.

If 4, x1, x2, x3, 28 are in AP then x3 = ?
A. 19

B. 23

C. 22

D. cannot be determined


Answer:

Here, first term = a = 4


Last term = l = 28


Number of terms = n = 5


l = a + (n - 1)d


⇒ 28 = 4 + (5 - 1)d


⇒ 28 - 4 = 4d


⇒ 4d = 24


⇒ d = 6


Therefore x3 = a + 3d


= 4 + 3(6)


= 4 + 18


= 22


Question 5.

If the nth term of an AP is (2n + 1), then the sum of its first three terms is
A. 6n + 3

B. 15

C. 12

D. 21


Answer:

Given: nth term = 2n + 1


∴ an = 2n + 1


Put n= 1, a1 = 3


Put n= 2, a2 = 5


Put n= 3, a3 = 7


Now, sum of first three terms = 3 + 5 + 7 = 15


Question 6.

The sum of first n terms of an AP is (3n2 + 6n). The common difference of the AP is
A. 6

B. 9

C. 15

D. - 3


Answer:

Let Sn denotes the sum of first n terms of an AP.


Sum of first n terms = Sn = 3n2 + 6n


Then nth term is given by: an = Sn - Sn - 1


∴ an = (3n2 + 6n) - [3(n - 1)2 + 6(n - 1)]


= (3n2 + 6n) - [3(n2 + 1 - 2n) + 6n - 6]


= 3n2 + 6n - 3n2 - 3 + 6n - 6n + 6


= 3 + 6n


Now, common difference = d = an - an - 1


= 3 + 6n - [3 + 6(n - 1)]


= 3 + 6n - 3 - 6n + 6


= 6


∴ Common difference = 6


Question 7.

The sum of first n terms of an AP is (5n – n2). The nth term of the AP is
A. (5 – 2n)

B. (6 – 2n)

C. (2n – 5)

D. (2n – 6)


Answer:

Let Sn denotes the sum of first n terms of an AP.


Sum of first n terms = Sn = 5n - n2


Then nth term is given by: an = Sn - Sn - 1


∴ an = (5n - n2) - [5(n - 1) - (n - 1)2]


= (5n - n2) - [5n - 5 - (n2 + 1 - 2n)]


= 5n - n2 - 5n + 5 + n2 + 1 - 2n


= 6 - 2n


Question 8.

The sum of first n terms of an AP is (4n2 + 2n). The nth term of this AP is
A. (6n – 2)

B. (7n – 3)

C. (8n – 2)

D. (8n + 2)


Answer:

Let Sn denotes the sum of first n terms of an AP.


Sum of first n terms = Sn = 4n2 + 2n


Then nth term is given by: an = Sn - Sn - 1


∴ an = (4n2 + 2n) - [4(n - 1)2 + 2(n - 1)]


= (4n2 + 2n) - [4(n2 + 1 - 2n) + 2n - 2]


= 4n2 + 2n - 4n2 - 4 + 8n - 2n + 2


= 8n - 2


Question 9.

The 7th term of an AP is – 1 and its 16th term is 17. The nth term of the AP is
A. (3n + 8)

B. (4n – 7)

C. (15 – 2n)

D. (2n – 15)


Answer:

Let a be the first term and d be the common difference.


Given: a7 = - 1


a16 = 17


Now, Consider a7 = - 1


⇒ a + 6d = - 1 …………(1)


Consider a16 = 17


⇒ a + 15d = 17 …………(2)


Subtract equation (1) from (2), we get,


9d = 18


⇒ d = 2


∴ Common difference = d = 2


Now, from equation (1), we get,


a = - 1 - 6d


= - 1 - 6(2)


= - 13


Now, nth term of the AP is given by


an = a + (n - 1)d


= - 13 + (n - 1)2


= 13 + 2n - 2


= 2n - 15


Question 10.

The 5th term of an AP is – 3 and its common difference is – 4. The sum of its first 10 terms is
A. 50

B. – 50

C. 30

D. – 30


Answer:

Let a be the first term and d be the common difference.


Given: a5 = - 3


Common difference = d = - 4


Now, Consider a5 = - 3


⇒ a + 4d = - 3


⇒ a + 4(-4) = - 3


⇒ a - 16 = - 3


⇒ a = 16 - 3


⇒ a = 13


Now, Sum of first n terms of an AP is


Sn = [2a + (n - 1)d]


∴ Sum of first 10 terms is given by:


S10 = [2(13) + (10 - 1)(-4)]


= 5[26 - 36]


= 5 × (-10)


= - 50


Question 11.

The 5th term of an AP is 20 and the sum of its 7th and 11th terms is 64. The common difference of the AP is
A. 4

B. 5

C. 3

D. 2


Answer:

Let a be the first term and d be the common difference.


Given: a5 = 20


a7 + a11 = 64


Now, Consider a5 = 20


⇒ a + 4d = 20 ……………(1)


Consider a7 + a11 = 64


⇒ a + 6d + a + 10d = 64


⇒ 2a + 16d = 64 …………(2)


Subtract twice of equation (1) from (2), we get,


8d = 24


⇒ d = 3


Question 12.

The 13th term of an AP is 4 times its 3rd term. If its 5th term is 16 then the sum of its first ten terms is
A. 150

B. 175

C. 160

D. 135


Answer:

Let a be the first term and d be the common difference.


Given: a13 = 4(a3)


a5 = 16


To find: Sum of first ten terms.


Now, Consider a13 = 4a3


⇒ a + 12d = 4[a + 2d]


⇒ a + 12d = 4a + 8d


⇒ 3a = 4d ………. (1)


Consider a5 = a + (5 - 1)d = 16


⇒ a + 4d = 16


⇒ a + 3a = 16 (from equation (1))


⇒ 4a = 16


⇒ a = 4 ………. (2)


∴ d = 3


Sum of n terms of an arithmetic series is given by:


Sn = [2a + (n - 1)d]


Therefore sum of 10 terms of the arithmetic series is given by:


∴ S10 = [2(4) + (10 - 1)(3)]


= 5 × [8 + 27]


= 5 × 35


= 175


Question 13.

An AP 5, 12, 19, ... has 50 terms. Its last term is
A. 343

B. 353

C. 348

D. 362


Answer:

Here, first term = 5


Common difference = 12 - 5 = 7


Given that there are 50 terms in the AP.


To find: Last term, i.e. 50th term = a50


Since an = a + (n - 1)d


∴ a50 = 5 + (50 - 1)7


= 5 + (49) × 7


= 5 + 343


= 348


Question 14.

The sum of first 20 odd natural numbers is
A. 100

B. 210

C. 400

D. 420


Answer:

Sum of first 20 odd natural numbers is 1 + 3 + 5 + 7 + … + 39.


This forms an arithmetic series with first term = a = 1


and common difference = d = 2


Sum of n terms of this arithmetic series is given by:


Sn = [2a + (n - 1)d]


Therefore sum of 20 terms of this arithmetic series is given by:


∴ S20 = [2(1) + (20 - 1)(2)]


= 10 [2 + 38]


= 10 × 40


= 400


Question 15.

The sum of first 40 positive integers divisible by 6 is
A. 2460

B. 3640

C. 4920

D. 4860


Answer:

First 40 positive integers divisible by 6 are 6, 12, 18, …, 240.


Sum of these numbers forms an arithmetic series 6 + 12 + 18 + … + 240.


Here, first term = a = 6


Common difference = d = 6


Sum of n terms of this arithmetic series is given by:


Sn = [2a + (n - 1)d]


Therefore sum of 40 terms of this arithmetic series is given by:


∴ S40 = [2(6) + (40 - 1)(6)]


= 20 [12 + 234]


=20 × 246


= 4920


Question 16.

How many two - digit numbers are divisible by 3?
A. 25

B. 30

C. 32

D. 36


Answer:

The two digit numbers divisible by 3 are 12, 15, 18, 21, …., 99.


This forms an AP with first term a = 12


and common difference = d = 3


Last term is 99.


Now, number of terms in this AP are given as:


99 = a + (n - 1)d


⇒ 99 = 12 + (n - 1)3


⇒ 99 - 12 = 3n - 3


⇒ 87 + 3 = 3n


⇒ 90 = 3n


⇒ n = 30


There are 30 two - digit numbers that are divisible by 3.


Question 17.

How many three - digit numbers are divisible by 9?
A. 86

B. 90

C. 96

D. 100


Answer:

The three digit numbers divisible by 9 are 108, 117, 126, …., 999.


This forms an AP with first term a = 108


and common difference = d = 9


Last term is 999.


Now, number of terms in this AP are given as:


999 = a + (n - 1)d


⇒ 999 = 108 + (n - 1)9


⇒ 999 - 108 = 9n - 9


⇒ 891 + 9 = 9n


⇒ 900 = 9n


⇒ n = 100


There are 100 three - digit numbers that are divisible by 9.


Question 18.

What is the common difference of an AP in which a18 – a14 = 32?
A. 8

B. – 8

C. 4

D. – 4


Answer:

Let a be the first term and d be the common difference.


Given: a18 – a14 = 32


⇒ (a + 17d) - (a + 13d) = 32


⇒ 17 d - 13d = 32


⇒ 4d = 32


⇒ d = 8


∴ d = common difference = 8


Question 19.

If an denotes the nth term of the AP 3, 8, 13, 18, ... then what is the value of (a30 - a20)?
A. 40

B. 36

C. 50

D. 56


Answer:

Here, First term = a = 3


Common difference = d = 8 - 2 = 5


To find: a30 – a20


So, nth term is given by:


an = a + (n - 1)d


∴ 30th term is:


a30 = 3 + (30 - 1) × 5


= 3 + 145


= 148


Now, 20th term is:


a20 = 3 + (20 - 1) × 5


= 3 + 95


= 98


Now, (a30 – a20) = 148 - 98


= 50


∴ (a30 – a20) = 50


Question 20.

Which term of the AP 72, 63, 54, ... is 0?
A. 8th

B. 9th

C. 10th

D. 11th


Answer:

In the given AP, the first term = a = 72


Common difference = d = 63 - 72 = - 9


To find: place of the term 0.


So, let an = 0


Since, we know that


an = a + (n - 1) × d


∴ 0 = 72 + (n - 1) × (-9)


⇒ - 72 = - 9n + 9


⇒ - 72 - 9 = - 9n


⇒ - 9n = - 81


⇒ n = 9


∴ 9th term of the AP is - 81.


Question 21.

Which term of the AP 25, 20, 15, ... is the first negative term?
A. 10th

B. 9th

C. 8th

D. 7th


Answer:

In the given AP, the first term = a = 25


Common difference = d = 20 - 25 = - 5


To find: place of first negative term.


So, an < 0


Since, we know that


an = a + (n - 1) × d


∴ 25 + (n - 1) × (-5) < 0


⇒ 25 - 5n + 5 < 0


⇒ - 5n + 30 < 0


⇒ - 5n < - 30


⇒ 5n > 30


⇒ n > 6


∴ 7th term of the AP is the first negative term.


Question 22.

Which term of the AP 21, 42, 63, 84, ... is 210?
A. 9th

B. 10th

C. 11th

D. 12th


Answer:

In the given AP, the first term = a = 21


Common difference = d = 42 - 21 = 21


To find: place of the term 210.


So, let an = 210


Since, we know that


an = a + (n - 1) × d


∴ 210 = 21 + (n - 1) × (21)


⇒ 210 = 21 + 21n - 21


⇒ 210 = 21n


⇒ n = 10


∴ 10th term of the AP is 210.


Question 23.

What is 20th term from the end of the AP 3, 8, 13, ..., 253?
A. 163

B. 158

C. 153

D. 148


Answer:

Here, First term = a = 3


Common difference = d = 8 - 3 = 5


Last term = l = 253


To find: 20th term from end.


So, nth term from end is given by:


an = l - (n - 1)d


∴ 20th term from end is:


a20 = 253 - (20 - 1) × 5


= 253 - 95


= 158


∴ 20th term from the end is 158.


Question 24.

(5 + 13 + 21 + + 181) =?
A. 2476

B. 2337

C. 2219

D. 2139


Answer:

Here, first term = 5


Common difference = d = 13 - 5 = 8


Last term = l = 253


To find: number of terms in the Arithmetic series


So, nth term is given by:


an = a + (n - 1)d


∴ 181 = 5 + (n - 1) × 8


⇒ 181 - 5 = 8n - 8


⇒ 176 = 8n - 8


⇒ 176 + 8 = 8n


⇒ 8n = 184


⇒ n = 23


Thus there are 23 terms in the arithmetic series.


Sum of first n terms of an AP is


Sn = [2a + (n - 1)d]


∴ Sum of 23 terms is given by:


S23 = [2(5) + (23 - 1)(8)]


= (23/2) × [10 + 176]


= (23/2) × 186


= 23 × 93


= 2139


Thus, sum of 23 terms of this Arithmetic series is 2139.


Question 25.

The sum of first 16 terms of the AP 10, 6, 2, ... is
A. 320

B. - 320

C. - 352

D. – 400


Answer:

Here, first term = 10


Common difference = d = 6 - 10 = - 4


Sum of first n terms of an AP is


Sn = [2a + (n - 1)d]


∴ S16 = [2(10) + (16 - 1)(-4)]


= 8 × [20 - 60]


= 8 × (-40)


= - 320


Thus, sum of 16 terms of this AP is - 320.


Question 26.

How many terms of the AP 3, 7, 11, 15, ... will make the sum 406?
A. 10

B. 12

C. 14

D. 20


Answer:

Here, first term = a =3


Common difference = d = 7 - 3 = 4


Let first n terms of the AP sums to 406.


∴ Sn = 406


To find: n


Now, Sn = (n/2) × [2a + (n - 1)d]


Since, Sn = 406


∴ (n/2) × [2a + (n - 1)d] = 406


⇒ (n/2) × [2(3) + (n - 1)4] = 406


⇒ (n/2) × [6 + 4n - 4] = 406


⇒ (n/2) × [(2 + 4n] = 406


⇒ n[1 + 2n] = 406


⇒ n + 2n2 = 406


⇒ 2n2 + n - 406 = 0


⇒ 2n2 - 28n + 29n - 406 = 0


⇒ 2(n - 14) + 29(n - 14)= 0


⇒ (2n + 29)(n - 14) = 0


⇒ n = 14 or n = - 29/2


∴ n= 14 (∵ n can’t be a fraction or negative number)


Question 27.

The 2nd term of an AP is 13 and its 5th term is 25. What is its 17th term?
A. 69

B. 73

C. 77

D. 81


Answer:

Given: a2 = 13


a5 = 25


To find: a17


Consider a2 = 13


⇒ a + d = 13 …………….(1)


Consider a5 = 25


⇒ a + 4d = 25 …………….(2)


Subtracting equation (1) from equation (2), we get,


3d = 12


⇒ d = 4


∴ Common difference = 4


From equation (1), we get


a = 13 - d


= 13 - 4


= 9


Thus a17 = a + 16d


= 9 + 16(4)


= 73


Question 28.

The 17th term of an AP exceeds its 10th term by 21. The common difference of the AP is
A. 3

B. 2

C. - 3

D. - 2


Answer:

|

Let a be the first term and d be the common difference.


Given: a17 = a10 + 21


To find: common difference = d


Consider a17 = a10 + 21


⇒ a + 16d = a + 9d + 21


⇒ 16d = 9d + 21


⇒ 16d - 9d = 21


⇒ 7d = 21


⇒ d = 3


∴ Common difference = 3


Question 29.

The 8th term of an AP is 17 and its 14th term is 29. The common difference of the AP is
A. 3

B. 2

C. 5

D. - 2


Answer:

Given: a8 = 17


a14 = 29


To find: common difference = d


Consider a8 = 17


⇒ a + 7d = 17 …………….(1)


Consider a14 = 29


⇒ a + 13d = 29 …………….(2)


Subtracting equation (1) from equation (2), we get,


6d = 12


⇒ d = 2


∴ Common difference = 2


Question 30.

The 7th term of an AP is 4 and its common difference is - 4. What is its first term?
A. 16

B. 20

C. 24

D. 28


Answer:

Given: a7 = 4


Common difference = d = - 4


To find: First term = a


Since, a7 = 4


⇒ a + 6d = 4


⇒ a + 6(-4) = 4


⇒ a = 4 + 24


⇒ a = 28